Test Bank – ACP: Dermatology

Cutaneous Manifestations of Systemic Diseases

1. A 56-year-old white man presents to you in clinic complaining of increasing shortness of breath. His symptom began several months ago and has gotten progressively worse over time. It is primarily exertional. On further review of systems, he reports chronic sinusitis, some fatigue, and a new nonpainful, nonpruritic rash on his lower extremities. His medical history is significant only for hypertension. On physical examination, you note nasal ulcerations and a mildly erythematous, papular rash with occasional nodules on his lower extremities. His pulmonary examination is notable for fine, bilateral crackles. Urine dipstick testing reveals proteinuria and hematuria. Among other tests, you order a blood test for cytoplasmic antineutrophil cytoplasmic antibodies (c-ANCA) and a chest x-ray. The chest x-ray reveals interstitial abnormalities bilaterally. A subsequent biopsy reveals a necrotizing granulomatous vasculitis. The c-ANCA is positive.

This patient’s findings are most consistent with which of the following diagnoses?
A. Lymphomatoid granulomatosis
B. Systemic lupus erythematosus (SLE)
C. Wegener granulomatosis
D. Churg-Strauss syndrome

Key Concept/Objective: To know the clinical presentation of Wegener granulomatosis

Wegener granulomatosis is associated with both distinctive and nonspecific mucocutaneous signs. Palpable purpura is one of the most common skin findings, but ulcers, papules, nodules, and bullae have also been described. In addition to upper and lower pulmonary symptoms, saddle-nose deformity, nasal ulcerations, and septal perforation should suggest the diagnosis of Wegener granulomatosis. Definitive diagnosis is made by the demonstration of a necrotizing granulomatous vasculitis in a patient with upper and lower respiratory tract disease and glomerulonephritis. ANCA autoantibodies are often present. The absence of asthma makes the diagnosis of Churg-Strauss syndrome unlikely. Although SLE would explain the skin rash, the abnormal urine dipstick result, and the progressive shortness of breath, the presence of nasal ulcerations, ANCA autoantibodies, and necrotizing granulomatous vasculitis would not be explained by SLE. Patients with lymphomatoid granulomatosis present with a predominance of pulmonary and nervous system manifestations; tests for ANCA autoantibodies are usually negative. (Answer: C— Wegener granulomatosis)

2. A 48-year-old African-American man with a history of diabetes and hypertension presents to your clinic complaining of a rash on his leg. It has been present for several months and is progressively getting worse. His diabetes is poorly controlled; diabetic complications include both chronic renal insufficiency and retinopathy. On physical examination, you note an 8 cm atrophic patch with a yellow central area and enlarging erythematous borders.

This patient’s symptoms are most consistent with which of the following cutaneous manifestations of diabetes?
A. Acanthosis nigricans
B. Necrobiosis lipoidica
C. Scleroderma
D. Erythrasma

Key Concept/Objective: To recognize necrobiosis lipoidica as a potential skin manifestation of diabetes

Necrobiosis lipoidica is a specific cutaneous manifestation of diabetes. Lesions consist of chronic atrophic patches with enlarging erythematous borders. The legs are most commonly affected. The center of lesions appears yellow because of subcutaneous fat that is visible through the atrophic dermis and epidermis. Occasionally, the lesions ulcerate. Necrobiosis lipoidica is often associated with diabetic nephropathy or retinopathy. Acanthosis nigricans has been reported in patients with insulin resistance syndrome; however, the lesions are velvety to the touch, are black in color, and are located predominantly in contact areas such as the axilla. Scleroderma causes subcutaneous infiltration of the skin, mostly in the trunk of patients with long-standing diabetes. Erythrasma is a fungal infection affecting the fifth intertriginous space. (Answer: B—Necrobiosis lipoidica)

3. A 29-year-old Japanese-American man comes to the emergency department complaining of a rash on his lower extremity. The rash began in a small area on his calf 1 day ago and has progressed rapidly. The patient has recently noticed erythematous streaks toward the groin. There is a large blisterlike lesion at the initial site. The patient also complains of fever and dizziness, which began a few hours ago. The patient’s medical history is significant only for hemachromatosis. He has lived in the United States since he was 10 years old. He lives alone and works as a sushi chef at a local restaurant. On physical examination, the patient is ill appearing. His temperature is 102.5° F (39.2° C), his heart rate is 120 beats/min, and his blood pressure is 90/40 mm Hg. His left lower extremity is notable for hemorrhagic bullous lesions and erythema tracking into the groin, with inguinal lymphadenopathy.

Which of the following organisms is most likely responsible for this patient’s condition?
A Neisseria meningitidis
B. Staphylococcus aureus
C. Borrelia burgdorferi
D. Vibrio vulnificus

Key Concept/Objective: To know the clinical presentation and risk factors for V. vulnificus infection

V. vulnificus infection can present as two distinct clinical syndromes: one arises from minor trauma sustained while swimming in lakes or the ocean or while cleaning seafood. Cellulitis occurs, with lymphangitis and bacteremia. In patients with hepatic cirrhosis, a systemic infection can occur after eating raw oysters. These patients develop hemorrhagic bullae, with leukopenia and disseminated intravascular coagulation (DIC). Other groups at risk for the systemic form of the disease are those with hemosiderosis, chronic alcohol abuse, and chronic liver disease other than cirrhosis. B. burgdorferi is the causative agent of Lyme disease; the characteristic rash is erythema chronicum migrans. The characteristic rash of N. meningitidis infection is petechiae, which in some patients progresses to purpura. Staphylococcal infections can produce bullous lesions, but these lesions are usually associated with severe cellulitis and are not usually hemorrhagic. (Answer: D—Vibrio vulnificus)

4. A 37-year-old white woman presents to you in clinic complaining of skin changes. She notes that the changes began on her face and hands but that they are becoming widespread. She complains that her skin feels tight and is beginning to change the appearance of her face. Physical examination confirms her report, and you note pursed lips and bound-down skin on her nose, creating a beaklike appearance. You suspect scleroderma and order several laboratory tests for further workup.

In patients with progressive systemic sclerosis, the presence of antibodies to which of the following portends a poor prognosis?
A. Scleroderma-70 (Scl-70)
B. Anticentromere antibody
C. Anti-Ro and anti-La
D. Antiphospholipids

Key Concept/Objective: To understand the significance of antibodies to Scl-70 in a patient with scleroderma

Progressive systemic sclerosis, also known as systemic scleroderma, is a frequently fatal disease in which patients present with Raynaud phenomenon and sclerodactyly (induration of the skin of the digits). Cutaneous induration can become widespread. Involvement of the face can lead to a characteristic appearance with pursed lips and bound-down skin of the nose that creates a beaklike appearance. Patients with antibodies to Scl-70 have a poor prognosis, often succumbing to renal disease and malignant hypertension. Patients with anticentromere antibodies have a more slowly progressive variant of scleroderma known as the CREST syndrome, which is characterized by cutaneous calcinosis, Raynaud phenomenon, esophageal dysmotility, sclerodactyly, and telangiectasia; with time, pulmonary fibrosis, pulmonary hypertension, and right-sided heart failure develop. Anti-Ro and antiLa antibodies are more commonly associated with Sjögren syndrome and SLE. The presence of antiphospholipid antibodies are characterized clinically by recurrent venous and arterial thrombosis, recurrent fetal loss, and prolongation of partial thromboplastin time; these findings are most likely to be seen in patients with SLE. (Answer: A—Scleroderma-70 [Scl-70])

For more information, see Lebwohl M: 2 Dermatology: I Cutaneous Manifestations of Systemic Diseases. ACP Medicine Online (www.acpmedicine.com). Dale DC, Federman DD, Eds. WebMD Inc., New York, July 2003

Papulosquamous Disorders

5. A 24-year-old man comes to your office with complaints of a diffuse, mildly pruritic rash that developed over the past 1 to 2 weeks. Examination reveals a papular eruption involving the trunk and extremities. You suspect pityriasis rosea.

Which of the following statements regarding the clinical features of pityriasis rosea is false?
A. Lesions typically occur on the trunk in a symmetrical fashion and form cleavage planes on the skin
B. The development of a herald-patch lesion 7 to 10 days before the onset of the diffuse eruption helps establish the diagnosis
C. Pityriasis rosea typically involves the palms and soles
D. The disorder is usually self-limited

Key Concept/Objective: To understand the distinguishing features of pityriasis rosea

Pityriasis rosea is a self-limited, exanthematous disease that manifests as oval papulosquamous lesions typically distributed in a symmetrical fashion over the trunk and extremities. The exact etiology is unclear, but viral triggers have been suggested. The eruption is usually preceded by a primary lesion consisting of a slightly raised, salmon-colored oval patch with fine scaling (the “herald patch”). Lesions tend to follow lines of cleavage on the skin and may appear on the back in a typical “fir tree” or “Christmas tree” distribution. The differential diagnosis of pityriasis rosea lesions includes secondary syphilis, tinea corporis, and tinea versicolor. The appearance of lesions on the palms and soles is more typical of secondary syphilis and may help distinguish this rash from pityriasis rosea. If there is high suspicion of syphilis or if the diagnosis is unclear, a serologic test for syphilis is warranted. The lesions of pityriasis rosea typically resolve spontaneously in 6 to 8 weeks. (Answer: C—Pityriasis rosea typically involves the palms and soles)

6. For an otherwise healthy individual with typical pityriasis rosea, which of the following would NOT be an appropriate option for treating symptoms

A. Low-potency topical steroids for lesions on the trunk
B. Oral antihistamines
C. Oral erythromycin
D. Use of ultraviolet B (UVB) or exposure to sunlight early in the course of the eruption
E. Retinoic acid

Key Concept/Objective: To understand the treatment options for pityriasis rosea

Pityriasis rosea is typically self-limited, but several treatment options exist. Foremost, patients should be reassured and educated about the benign nature of the disease. If pruritis is a prominent symptom, oral antihistamines are usually effective. Low-potency topical steroids have been shown to be of benefit. Exposure to ultraviolet light has been shown to shorten the duration of the eruption, especially if treatment is started within the first week of onset. A single trial demonstrated that a 14-day course of oral erythromycin was safe and led to complete resolution of the eruption within 2 weeks in a third of patients. Retinoic acid has not commonly been employed to treat pityriasis rosea. (Answer: E—Retinoic acid)

7. A 40-year-old man comes to clinic complaining of an itchy rash on his wrist and hands, which he first noted 3 weeks ago. He has a history of hypertension but has otherwise been healthy. Skin examination reveals several small, polygonal papules on the dorsa of both hands and on the flexor surface of the left wrist. A few of the wrist lesions appear in a linear distribution. You also notice a papular lesion on the buccal mucosa; a white, lacy pattern appears on the surface of the lesion.

Which of the following is the most likely diagnosis?
A. Psoriasis
B. Lichen planus
C. Tinea versicolor
D. Pityriasis rosea
E. Candidiasis

Key Concept/Objective: To know the typical appearance of lichen planus

Lichen planus is a localized or generalized inflammatory mucocutaneous eruption consisting of violaceous, flat-topped, polygonal papules; it occurs most commonly in patients 30 to 60 years of age. The etiology is unknown, but a variety of drugs (e.g., beta blockers, methyldopa, and nonsteroidal anti-inflammatory drugs) have been reported to cause lichenoid reactions in the skin. Common sites of involvement include the flexor surfaces of the wrists, the dorsal surfaces of the hands, the sacrum, oral mucous membranes, and the genitalia. Mucous membrane lesions typically have a white, reticulated appearance on the papule surface (Wickham striae), which helps establish the diagnosis. Linear lesions that appear in areas of local skin trauma (the Koebner phenomenon) are also typical. Patients usually complain of mild itching. Over 50% of patients with cutaneous lesions experience involvement of the oral mucosa. This finding, along with the typical appearance and distribution of the lesions, helps distinguish it from the other conditions listed. Patients with acute lichen planus have a good prognosis, but the chronic form may last for several years. Treatment generally consists of emollients and topical steroids, but systemic steroids may also be of use. A corticosteroid in a vehicle that adheres to the mucosal surface (e.g., Orabase) is useful for treating mouth lesions. (Answer: B—Lichen planus)

8. A 59-year-old man with long-standing psoriasis has had a recent worsening of his disease. About 2 weeks ago, several new psoriatic lesions developed; this was followed by diffuse skin involvement with erythema and subsequent scaling. The patient complains of skin tightness, pruritus, fever, and malaise of 2 days’ duration. You suspect an exfoliative erythroderma reaction.

Which of the following statements regarding this patient’s condition is true

A. The majority of cases are spontaneous and are not associated with an underlying skin condition or systemic illness
B. Dehydration and high-output cardiac failure secondary to transepidermal water loss can occur
C. Bacterial infection is the likely cause
D. The condition most commonly occurs in females younger than 20 years

Key Concept/Objective: To understand that erythroderma is a potentially serious generalized skin reaction that can occur in patients with underlying dermatologic or systemic disease

This patient has exfoliative erythroderma, which is a generalized scaling erythematous dermatitis involving all or almost all of the cutaneous surface. The condition most commonly occurs in patients with underlying dermatoses such as psoriasis, atopic dermatitis, drug eruptions, or contact dermatitis. It may occur in patients with other systemic illnesses, including lymphoma and leukemia. It most commonly occurs in patients older than 40 years and is twice as common in men than in women. In severe cases, hospitalization may be necessary to treat significant fluid losses that occur as a result of disruption of the skin barrier. Treatment includes restoration of fluid and electrolytes and prevention of large, insensible fluid losses through the skin. Wet dressings applied over intermediatestrength topical steroid preparations are useful. Although infection is not the primary cause of the disorder, there is substantial risk of secondary skin infection, which may warrant the use of systemic antibiotics. (Answer: B—Dehydration and high-output cardiac failure secondary to transepidermal water loss can occur)

For more information, see Abel EA: 2 Dermatology: II Papulosquamous Disorders. ACP Medicine Online (www.acpmedicine.com). Dale DC, Federman DD, Eds. WebMD Inc., New York, June 2002

Psoriasis

9. A 26-year-old woman presents to your primary care clinic for the evaluation of a rash. The patient has no significant medical history and is not taking any medications. The family history is unrevealing. Physical examination reveals sharply demarcated, erythematous scaling plaques at both elbows and knees. Examination of the fingernails reveals groups of tiny pits. A presumptive diagnosis of psoriasis is made.

Which of the following statements regarding the epidemiology of psoriasis is accurate?
A. Psoriasis occurs more commonly in women than in men
B. On average, onset of psoriasis occurs during the fifth decade
C. In northern latitudes, exacerbations of psoriasis commonly occur during the spring and summer
D. In persons with earlier age of onset, psoriasis is more likely to be severe

Key Concept/Objective: To understand the epidemiology of psoriasis

The estimated prevalence of psoriasis ranges from 0.5% to 4.6% worldwide. The reasons for the geographic variation in prevalence are unknown, but climatic factors and genetics may play a role. On the basis of a survey mailed to 50,000 households, the prevalence in the United States is estimated to be 2.6%. Psoriasis can occur in patients of any age, with some cases being reported at birth and others being reported in patients older than 100 years. The average age of onset is 23 years in the United States. Psoriasis occurs with equal frequency in men and women. In populations in which there is a high prevalence of psoriasis, onset tends to occur at an earlier age. In persons with earlier age of onset, psoriasis is more likely to be severe, with involvement of a large area of skin surface. It has long been known that psoriasis improves when patients are exposed to sunny climates and to regions of lower latitude. In northern latitudes, exacerbation of psoriasis commonly occurs during the fall and winter. (Answer: D—In persons with earlier age of onset, psoriasis is more likely to be severe)

10. An 18-year-old college student presents to the student health clinic for evaluation of a rash. The patient was recently evaluated for sore throat and diagnosed with streptococcal pharyngitis. Physical examination reveals small, scaling erythematous papules on the trunk and bilateral extremities.

This patient’s clinical presentation is most consistent with which clinical variant of psoriasis?
A. Plaque-type
B. Guttate
C. Erythrodermic
D. Pustular

Key Concept/Objective: To be able to differentiate among the clinical variants of psoriasis

Nearly 90% of psoriasis patients have plaque-type psoriasis, a form that is characterized by sharply demarcated, erythematous scaling plaques. The elbows, knees, and scalp are the most commonly affected sites. The intergluteal cleft, palms, soles, and genitals are also commonly affected, but psoriasis can involve any part of the body. Lesions frequently occur in a symmetrical pattern of distribution. Many patients have only one or a few lesions that persist for years and that occasionally resolve after exposure to sunlight. Other patients can be covered with plaques that become confluent, affecting nearly 100% of the body surface area. Nail involvement is common, particularly in patients with severe disease. The second most common form of psoriasis, guttate psoriasis, affects fewer than 10% of patients and is characterized by the development of small, scaling erythematous papules on the trunk and extremities. This form of psoriasis often occurs after streptococcal infection. Patients with plaque-type psoriasis can develop guttate psoriasis. Conversely, patients with guttate psoriasis frequently develop plaque-type psoriasis.
Erythrodermic psoriasis is a severe form of psoriasis that often affects the entire cutaneous surface. Patients present with an exfoliative erythroderma in which the skin is very red and inflamed and is constantly scaling. Patients are acutely ill, their skin having lost all protective function. Loss of temperature control, loss of fluids and nutrients through the impaired skin, and susceptibility to infection make this a life-threatening condition. Some patients present with erythrodermic psoriasis de novo; others develop erythrodermic psoriasis after having typical plaque-type or guttate psoriasis. Erythrodermic psoriasis can occur after withdrawal of systemic corticosteroids, after phototherapy burns, as a result of antimalarial treatment, as a result of a drug-induced hypersensitivity reaction, or for no apparent reason. Erythrodermic psoriasis has been associated with cutaneous T cell lymphoma.
Pustular psoriasis, another severe form of the disease, can occur in patients with preexisting psoriasis, or it can arise de novo. Pustular psoriasis can be generalized (von Zumbusch-type) or localized to the palms and soles. In either case, the condition is severe and debilitating. In generalized pustular psoriasis, the body is covered with sterile pustules. As with erythrodermic psoriasis, the protective functions of the skin are lost, and patients may succumb to infection or hypovolemia and electrolyte imbalance caused by loss of fluid through the skin. Although fever and leukocytosis are commonly found in patients with pustular psoriasis, the possibility of infection should not be overlooked; patients with pustular psoriasis have died of staphylococcal sepsis. As with erythrodermic psoriasis, pustular psoriasis is most commonly precipitated by withdrawal of systemic corticosteroids, but it can also result from therapy with antimalarial drugs or lithium, and it can develop spontaneously. (Answer: B—Guttate)

11. A 32-year-old woman presents to clinic to establish primary care. She has recently relocated from another city. The patient’s medical history is significant for psoriasis, for which she has been treated with methotrexate, 20 mg a week for 6 years.

Which of the following should NOT be done to monitor for methotrexate toxicity?
A. Monitoring of liver function tests (LFTs)
B. Liver biopsy
C. Monitoring of the complete blood cell count (CBC)
D. Bone marrow biopsy

Key Concept/Objective: To understand monitoring for methotrexate toxicity in patients with psoriasis

The antimetabolite methotrexate was considered effective for the treatment of psoriasis because of its antimitotic effect on proliferating keratinocytes. However, tissue culture studies have suggested that activated lymphoid cells in the lymph nodes, blood, and skin are a likely target of methotrexate; proliferating macrophages and T cells are 100 times more sensitive to methotrexate than proliferating epithelial cells. These findings may be relevant to the mechanism of action of methotrexate in other immunologically based disorders, including psoriatic arthritis, rheumatoid arthritis, and Crohn disease. Methotrexate is best given in a single weekly oral dose of up to 30 mg or in three divided doses at 12hour intervals during a 24-hour period (e.g., at 8:00 A.M., at 8:00 P.M., and again at 8:00 A.M.). Side effects of methotrexate therapy include bone marrow suppression, nausea, diarrhea, stomatitis, and hepatotoxicity. Methotrexate is teratogenic and can cause reversible oligospermia. Evaluation by tests of liver function, renal function, and blood elements must be made before and throughout the course of methotrexate therapy. Cases of pancytopenia after low-dose methotrexate therapy underscore the hazards of use of the drug in patients with renal insufficiency or in patients who are concomitantly receiving drugs that increase methotrexate toxicity.
The use of liver biopsy to monitor patients on methotrexate has been a source of great controversy. Liver biopsies are not routinely performed in patients with rheumatoid arthritis who are undergoing treatment with methotrexate, but liver biopsy has been advocated in patients with psoriasis. Patients with psoriasis who are treated with methotrexate are more prone to hepatic fibrosis, possibly because of their underlying disease or because of the concomitant treatments they are given. Current guidelines call for the use of liver biopsy in patients with psoriasis who have received a cumulative dose of 1 to 1.5 g of methotrexate and who do not have a history of liver disease or alcoholism. Biopsy should be performed early in the course of treatment in patients with a history of hepatitis C, alcoholism, or other liver disease. Risk factors for hepatotoxicity include heavy alcohol intake, obesity, a history of diabetes or hepatitis, and abnormal results on liver function testing.
Although methotrexate causes bone marrow suppression, routine bone marrow biopsies are not indicated. Close monitoring with a monthly CBC is needed. (Answer: D—Bone marrow biopsy)

12. A 32-year-old high school teacher reports a mildly itchy new rash over the past week. He has been generally healthy, although he did take a course of penicillin for culture-positive streptococcal pharyngitis several weeks ago. He does not smoke, drinks alcohol only occasionally, and has been monogamous with his wife over the 5 years they have been married. He has had no fever, chills, eye symptoms, anorexia, nausea, diarrhea, bloody stool, abdominal pain, penile sores or discharge, dysuria, or joint pains. On examination, the patient is afebrile, with multiple sharply demarcated scaly papules 3 to 10 mm in diameter distributed symmetrically on his trunk, arms, palms, and penis. There are no target lesions or oral lesions, and no lymphadenopathy is found.

What is the most likely cause of this patient’s rash?
A. Primary HIV infection
B. Secondary syphilis
C. Reiter syndrome
D. Guttate psoriasis
E. Drug reaction

Key Concept/Objective: To be able to recognize guttate psoriasis

This is a classic presentation of guttate psoriasis, with onset after a recent streptococcal infection; a symmetrical distribution involving trunk, extremities, palms, and penis; and well-demarcated, small, scaly, erythematous papules. In contrast, the rash of primary HIV infection is a maculopapular, diffuse eruption, with poorly defined borders and no scaling, usually accompanied by low-grade fever, malaise, lymphadenopathy, and other flulike symptoms. Secondary syphilis can cause a scaly rash that may include the palms and soles, but the rash is not itchy and is usually accompanied by lymphadenopathy and/or oral lesions. Secondary syphilis is also accompanied by a positive rapid plasma reagin test. Reiter syndrome usually presents as a tetrad of arthritis, urethritis, conjunctivitis/uveitis, and mucocutaneous lesions. The skin and nail lesions of Reiter syndrome can be difficult to distinguish clinically from psoriasis. For example, the balanitis of Reiter syndrome can look scaly or pustular as in psoriasis; the keratoderma blennorrhagicum can cause a scaly or pustular rash on the palms or soles that can be indistinguishable from psoriasis; and Reiter syndrome nail changes (ridging, pitting, onycholysis) can mimic psoriasis. Though this patient does have penile lesions that could be confused with balanitis, he does not have the rest of the tetrad of symptoms. Most patients with Reiter syndrome also describe a preceding diarrheal illness or sexually transmitted infection, which this patient did not report. Drug reactions usually occur sooner after use of antibiotics than was seen in this patient, who took penicillin several weeks before his rash developed. Additionally, though drug reactions are often symmetrical, they are usually more diffuse, maculopapular eruptions and are worse on the trunk than on the extremities. If scaly at all, a drug reaction is then usually diffuse, involving the entire skin surface with an erythroderma or exfoliative dermatitis rather than with the small, discrete papules seen in this patient. (Answer: D— Guttate psoriasis)

13. A 62-year-old man wishes to continue oral therapy begun by another physician for onychomycosis. He reports disfigured nails over the past year, and he began daily oral ketoconazole 2 months ago without much change in the appearance of his nails. His past medical history includes hyperuricemia, chronic neck and back pain, sciatica, and excessive alcohol ingestion. On review of systems, he also reports finger stiffness and pain, especially in the distal interphalangeal (DIP) joints of multiple fingers. On examination, the patient’s fingers are red, tender, and slightly swollen, appearing somewhat sausage-shaped, without obvious synovitis. Multiple fingernails show deep longitudinal ridging with some pitting, thickening, yellowish discoloration, and onycholysis. The distal ends of multiple fingers are encased in heaped-up scale, debridement of which reveals necrotic tissue underneath. No lymphadenopathy is found in the neck, axillae, or epitrochlear areas. Laboratory testing shows mild anemia with a normal white blood cell count; negative rheumatoid factor, rapid plasma reagin, and antinuclear antibody tests; and a mildly elevated uric acid and ESR. Hand x-rays show erosions in some DIP joints without hyperostosis or bone-cyst formation.

Which of the following is the most likely diagnosis for this patient?
A. Chronic gout
B. Osteoarthritis
C. Psoriatic arthritis
D. Rheumatoid arthritis
E. Systemic lupus erythematosus (SLE)

Key Concept/Objective: To be able to recognize psoriatic arthritis

Arthritis involving DIP joints and associated with characteristic psoriatic fingernail changes, including pitting, yellow discoloration, onycholysis, ridging, and subungual hyperkeratosis, is most likely to be psoriatic arthritis. Elevated uric acid and mild iron deficiency anemia can also accompany psoriasis because of high skin turnover. Careful examination of this patient’s scalp, umbilicus, gluteal fold, and groin may reveal more characteristic scaly plaques. Chronic gouty arthritis can occur with chronic elevations of uric acid, but it is usually accompanied by tophi, seen on examination as gross deformities in or near the affected joints, and punched-out erosions with overhanging cortical bone (also called “rat-bite” lesions) adjacent to tophaceous deposits, seen on x-ray: findings that are not present in this patient. Additionally, gouty arthritis would not explain the skin findings. Osteoarthritis, like psoriatic arthritis, can affect DIP joints; but unlike psoriatic arthritis, osteoarthritis will cause Heberden nodes at the DIP joints and will display x-ray findings of hyperostosis and sometimes bone-cyst formation. Rheumatoid arthritis usually spares the DIP joints and causes a spongy swelling of synovial tissue at the metacarpophalangeal and/or wrist joints. SLE can present with rash and joint problems. Unlike the rash of psoriasis, that of SLE characteristically appears on the face or other sunexposed areas and produces localized red plaques, follicular plugging, atrophy, and telangiectasias: quite unlike the isolated periungual scaling seen in this patient. The arthropathy of SLE does not cause bony erosions. (Answer: C—Psoriatic arthritis)

14. A 23-year-old man presents with worsening pain and swelling in his right ankle, which he has had for the past month. He is otherwise healthy, though he admits to an unhealthy lifestyle, including nightshift work, heavy alcohol use on the weekends, and occasional unprotected sex with men, though he has had none in the past 12 months. He denies any history of sexually transmitted disease or intravenous drug use, diarrheal illness, fever, chills, weight loss, dysuria, penile discharge, or other joint pains. On examination, he is in no apparent distress. Removal of his baseball cap reveals a 6 × 15 cm patch of a sharply demarcated, erythematous, scaly rash on his anterior scalp and forehead. Skin and nail examination reveals no further rashes in the groin, gluteal fold, or umbilicus and no nail pitting. A few 1 cm nodes are found in his neck and groin. His right ankle is normal in color but swollen and boggy, with decreased range of motion and mild tenderness to palpation. Biopsy of his scalp suggests psoriasis.

What other testing should be performed on this patient?

A. HIV test
B. Uric acid level
C. Urethral, anal, and pharyngeal swabs for gonorrhea
D. HLA-B27 test
E. Antinuclear antibody test

Key Concept/Objective: To understand that new psoriasis can be associated with HIV infection, especially in patients who have risk factors for sexually transmitted disease or who have examination findings that suggest HIV infection

This patient is presenting with new-onset psoriasis and likely psoriatic arthritis. He also has a risk factor for HIV infection (unprotected sex with men), as well as unexplained lymphadenopathy. More careful examination may reveal other clues to HIV infection, including oral candidiasis, oral hairy leukoplakia, gingivitis, or seborrheic dermatitis. HIV testing is most wise in this case. Though gout can affect the ankle joint, there is usually much greater pain and inflammation associated with acute gouty arthritis. Furthermore, uric acid levels are not always helpful in making a diagnosis of gout, because they can be normal in up to 25% of patients with gout. Swabs for gonorrhea can be helpful in diagnosing septic arthritis, though as with gout, the involved joint is usually more obviously inflamed (warm, red, very tender to any movement). HLA-B27 testing has no role in the diagnosis or treatment of psoriatic arthritis. The HLA-B27 test is often positive in patients with Reiter syndrome, although this patient does not have the classic tetrad of symptoms (see Question 12). Antinuclear antibody test is not warranted, because this patient does not have any symptoms or findings that suggest SLE. (Answer: A—HIV test)

For more information, see Abel EA: 2 Dermatology: III Psoriasis. ACP Medicine Online
(www.acpmedicine.com). Dale DC, Federman DD, Eds. WebMD Inc., New York, June 2002

Eczematous Disorders, Atopic Dermatitis, Ichthyoses

15. A 35-year-old woman presents to clinic complaining of a pruritic rash on her hands. She denies recently changing the detergents and soaps she uses, and she does not wear jewelry. She admits to being under a great deal of stress at work over the past few days. Examination is notable for small, clear vesicles on the sides of her fingers, and there is associated evidence of excoriation.

What is the most likely diagnosis for this patient

A. Nummular eczema
B. Contact dermatitis
C. Dyshidrotic eczema (pompholyx)
D. Seborrheic dermatitis

Key Concept/Objective: To know the differential diagnosis of eczematous disorders and to recognize the presentation of dyshidrotic eczema

Eczema is a skin disease characterized by erythematous, vesicular, weeping, and crusting patches associated with pruritus. The term is also commonly used to describe atopic dermatitis. Examples of eczematous disorders include contact dermatitis, seborrheic dermatitis, nummular eczema, and dyshidrotic eczema (pompholyx). Contact dermatitis is very common and can be induced by allergic or irritant triggers. The distribution of the rash in contact dermatitis coincides with the specific areas of skin that were exposed to the irritant (e.g., in patients sensitive to nickel, rashes may appear on fingers on which rings containing nickel are worn; in patients sensitive to detergent, rashes may appear on areas covered by clothing containing detergent). This patient does not have a history of any particular exposure, and the rash occurs only on the sides of several fingers. Seborrheic dermatitis is also common and is characterized by involvement of the scalp, eyebrows, mustache area, nasolabial folds, and upper chest. Nummular eczema is characterized by coinshaped patches occurring in well-demarcated areas of involvement. This patient most likely suffers from dyshidrotic eczema, which tends to occur on the side of the fingers, is intensely pruritic, and often flares during times of stress. The treatment includes compresses, soaks, antipruritics, and topical steroids. Severe cases may require systemic steroids. (Answer: C—Dyshidrotic eczema [pompholyx])

16. A 21-year-old man presents to the acute care clinic complaining of itching. He states that since childhood, he has had a recurrent rash characterized by “red, itchy, dry patches” of skin. Sometimes the rash is associated with “little bumps.” Examination of his skin reveals erythematous, scaling plaques on the flexural surfaces of his arms with associated excoriations. You suspect the patient has atopic dermatitis.

Of the following findings, which is NOT among the major diagnostic criteria of atopic dermatitis?
A. Personal or family history of atopy
B. Pruritus
C. Chronic or chronically recurring dermatosis
D. Elevated serum IgE level

Key Concept/Objective: To recognize the clinical presentation of atopic dermatitis and to know the major diagnostic criteria

Atopic dermatitis is a clinical diagnosis. The major diagnostic criteria for atopic dermatitis include a personal or family history of atopy (including asthma, allergic rhinitis, allergic conjunctivitis, and allergic blepharitis); characteristic morphology and distribution of lesions (usually eczematous patches in flexural areas in adults and extensor surfaces in children who crawl; lichenification can occur with nodule formation in chronic cases); pruritus (virtually always present); and a chronic or chronically recurring course. An elevated serum IgE level is not a major diagnostic criterion. This patient is somewhat unusual in that his childhood atopic dermatitis has persisted past puberty (this occurs in only 10% to 15% of cases). (Answer: D—Elevated serum IgE level)

17. An 18-year-old woman presents for treatment of chronic dry skin and scaling. The rash typically involves the extensor surfaces of her extremities. She notes that she has had this condition since infancy and that her father has it as well. Her medical records indicate that she has been diagnosed with ichthyosis vulgaris by a dermatologist.

Which of the following statements is false?
A. The ichthyoses are a group of diseases characterized by abnormal cornification of the skin leading to excessive scaling
B. Ichthyosis can be an acquired condition associated with endocrinopathies, autoimmune diseases, HIV infection, lymphomas, and carcinomas
C. The most common form of ichthyosis is acquired ichthyosis
D. Treatment of ichthyosis includes emollients and keratolytics

Key Concept/Objective: To know the presentation of ichthyosis vulgaris and to be familiar with the ichthyoses

Etiologies of the ichthyoses are diverse, but the ichthyoses share common manifestations and treatments. The most common form is ichthyosis vulgaris, which is inherited in an autosomal dominant fashion (as seen in this patient); disease onset occurs in patients 3 to 12 months of age. Other forms of ichthyoses include recessive X-linked ichthyosis, lamellar ichthyosis, congenital ichthyosiform erythroderma, epidermolytic hyperkeratosis, and acquired ichthyosis. Acquired ichthyosis is associated with multiple disorders, including HIV infection and endocrinopathies; it can also occur as a paraneoplastic syndrome that is usually associated with lymphomas and carcinomas. Epidermolytic hyperkeratosis is the most difficult form to treat because therapeutic agents can induce blistering. Standard therapies are emollients (such as petrolatum) and keratolytics (such as lactic acid with or without propylene glycol). Antimicrobial agents are also frequently used to combat the odor and other complications of bacterial colonization of the affected skin. (Answer: C—The most common form of ichthyosis is acquired ichthyosis)

For more information, see Stevens SR, Cooper KD, Kang K: 2 Dermatology: IV Eczematous Disorders, Atopic Dermatitis, and Ichthyoses. ACP Medicine Online (www.acpmedicine.com). Dale DC, Federman DD, Eds. WebMD Inc., New York, March 2002

Contact Dermatitis and Related Disorders

18. A 45-year-old woman with a history of malaise, weight loss, and recurrent upper respiratory infections of several months’ duration was picnicking with friends. Within a few minutes, most of the picnickers realized that they were developing a pruritic, erythematous skin reaction on skin exposed to the lush ground cover around them. One person belatedly recognized the plant as poison ivy. The patient, however, did not develop a reaction.

What is the most likely reason for this patient’s failure to develop a reaction?
A. She had anergy related to weight loss
B. She had impaired T cell immunity related to as yet undiagnosed AIDS
C. She had not been previously exposed to the allergen found in poison ivy
D. She was wearing sunscreen, which formed a barrier to the allergen
E. Before the picnic, she had taken a dose of diphenhydramine for her persistent rhinorrhea

Key Concept/Objective: To understand immediate-type sensitivity and the role that a history of previous exposure to an allergen plays in making a diagnosis

This woman did not experience an immediate reaction to poison ivy, which is a type I immediate hypersensitivity reaction mediated by circulating antibodies; in this case, the reaction would have been to toxicodendron antigens associated with poison ivy, poison oak, and sumac. Although it is possible that the patient had impaired immunity secondary to weight loss or possibly to HIV infection, even in these cases the immediate type I hypersensitivity reaction would likely occur. A barrier cream would not protect a patient from a contact dermatitis; the cream would only be an additional vector for the spread of the hapten or allergen. Although a dose of diphenhydramine before exposure to an allergen may diminish an allergic reaction, the primary reaction would still occur. (Answer: C— She had not been previously exposed to the allergen found in poison ivy)

19. A 30-year-old salesman in a party supply store that specializes in balloons develops a severe pruritic erythematous diffuse skin reaction after eating avocado.

What is the most likely explanation for this patient’s reaction?
A. Delayed type IV reactivity to the avocado as the primary allergen
B. Immediate type I reactivity to the avocado as the primary allergen
C. Irritant contact dermatitis reaction to the avocado
D. Immediate type I cross-reactivity reaction to the avocado with primary latex allergy as the underlying allergic cause
E. IgM-mediated allergic reaction

Key Concept/Objective: To understand natural rubber and latex allergy and cross-reactivity with certain fruits, including avocados, chestnuts, kiwi, and bananas

The patient has a history of exposure to latex through his work with balloons, and he develops a systemic pruritic reaction after eating avocado. The immediate reaction time rules out a type IV reaction, which usually takes 12 to 48 hours to occur. This patient would not be having an irritant reaction, because his exposure reaction is diffuse, not focal, as would be the case with irritant contact dermatitis. The latex allergic reaction is mediated through IgE, not IgM. (Answer: D—Immediate type I cross-reactivity reaction to the avocado with primary latex allergy as the underlying allergic cause)

20. What is the best method of distinguishing irritant contact dermatitis from allergic contact dermatitis?
A. History of exposure frequency
B. Examination of clinical features and distribution of rash
C. Patch testing
D. Histologic evaluation of skin biopsy of rash
E. Family and travel history

Key Concept/Objective: To understand the value of patch testing in distinguishing irritant contact dermatitis from allergic contact dermatitis

Determining the etiology of a contact dermatitis is difficult. The gold standard is patch testing, although its sensitivity and specificity vary with the tested allergen. Although history of exposure and examination of the distribution and quality of the reaction are valuable, the best method of distinguishing a contact irritant reaction from an allergic contact reaction is patch testing. The histologies of contact irritant dermatitis and allergic dermatitis are identical, and therefore, histologic evaluation would not be useful in determining whether a reaction is allergy-related. (Answer: C—Patch testing)

21. A 43-year-old woman with a long history of chronic actinic dermatitis was experiencing frequent upper respiratory infections, weight loss, and malaise. She was pleased that her chronic actinic dermatitis was improving to the point of being nearly resolved but was concerned about her recurrent infections and weight loss. She presented to a physician, who diagnosed HIV in her blood; the patient had a high viral load count and a very low helper T cell count. The physician started her on didanosine (DDI), zidovudine (AZT), and indinavir. A week later, the patient felt better, but there was evidence of her rash recurring in its previous pattern of distribution.

What is the probable reason for the recurrence of this patient’s dermatitis?
A. Allergic reaction to DDI
B. Allergic reaction to AZT, DDI, and the antiretroviral therapy
C. Cytomegalovirus dermatitis
D. T cell–mediated dermatitis
E. Kaposi sarcoma manifesting in the same distribution pattern as the previous chronic actinic dermatitis

Key Concept/Objective: To understand the important role of the T cell in the pathophysiology of chronic actinic dermatitis

In this case, a woman with a chronic actinic dermatitis mediated by type IV hypersensitivity via the T cells is compromised through development of AIDS. After therapy that restores her T cell function, she again develops dermatitis. A cytomegalovirus dermatitis would not have the same pattern, type, and distribution as chronic actinic dermatitis. Kaposi sarcoma would be more focal and distinctly pigmented, very unlike chronic contact dermatitis. Although allergic dermatitis can often be seen in the sites of previous skin trauma, such a reaction to the HIV drugs at a time of impaired T cell immunity would be hard to explain, especially in the absence of previous exposure to the medications. (Answer: D—T cell–mediated dermatitis)

22. A 24-year-old female emergency department nurse with a history of spina bifida with resultant neurogenic bladder and spastic paresis of the legs died of anaphylactic shock after intercourse during which her partner used a condom.

Which of the following is the most likely cause of this patient’s shock?
A. Peritonitis after rupture of the bladder
B. Allergic anaphylactic reaction to latex in her partner’s condom
C. Allergic anaphylactic reaction to a meal consumed less than an hour before death
D. Blood loss associated with retroperitoneal hemorrhage for which she was predisposed because of the underlying spina bifida
E. Sepsis from a urinary tract infection (UTI)

Key Concept/Objective: To be able to recognize latex allergy

Risk factors for development of a natural rubber and latex (NRL) allergy include exposure through the workplace, which in this case involves use of latex gloves in the emergency department. Spina bifida patients have a 30% to 65% prevalence of NRL allergy. Cross-sensitization to NRL through exposure to chestnuts, kiwi, bananas, or avocado is also recognized. The reaction is mediated by IgE and the T cell. In this case, the latex in the partner’s condom was the most likely allergen to cause anaphylaxis. (Answer: B—Allergic anaphylactic reaction to latex in her partner’s condom)

For more information, see Taylor JS: 2 Dermatology: V Contact Dermatitis and Related Disorders. ACP Medicine Online (www.acpmedicine.com). Dale DC, Federman DD, Eds. WebMD Inc., New York, March 2001

Cutaneous Adverse Drug Reactions

23. A 23-year-old man presented with fever and sore throat; physical examination revealed an erythematous oropharynx and cervical lymphadenopathy. The patient had no known history of drug allergy. He was started on an empirical regimen of amoxicillin for streptococcal pharyngitis. Three days later, he returned to your office complaining that his symptoms had continued and that he had developed a rash. An erythematous maculopapular rash was noted on physical examination. A monospot test was performed. The results come back positive.

Which of the following statements regarding this patient’s exanthematous drug eruption is true?
A. Persistence of fever is not helpful in determining whether the symptoms are the result of an allergic reaction, because fever is common in simple exanthematous eruptions
B. Systemic corticosteroids are always required to treat this drug eruption
C. After the patient’s infectious process resolves, he will be able to tolerate all ß-lactam antibiotics, including ampicillin
D. In patients with viral infections, the mechanism of exanthematous eruption caused by ampicillin is IgE-mediated mast cell degranulation
E. This patient’s rash can be expected to progress to a vesicular stage before resolution

Key Concept/Objective: To understand that ampicillin-amoxicillin–related exanthematous eruptions that occur in patients with viral infections do not appear to be IgE-mediated and that patients can tolerate penicillins and cephalosporins once the infection resolves

The etiology of the ampicillin rash that occurs in association with a viral infection is unknown, but the rash does not appear to be IgE-mediated. Patients can tolerate all ß-lactam antibiotics, including ampicillin, once the infectious process has resolved. Fever is not associated with simple exanthematous eruptions. These eruptions usually occur within 1 week after the beginning of therapy and generally resolve within 7 to 14 days. The exanthem’s turning from bright red to brownish red marks resolution. Resolution may be followed by scaling or desquamation. The treatment of simple exanthematous eruptions is generally supportive. For example, oral antihistamines used in conjunction with soothing baths may help relieve pruritus. Topical corticosteroids are indicated when antihistamines do not provide relief. Systemic corticosteroids are used only in severe cases. Discontinuance of the offending agent is recommended. (Answer: C—After the patient’s infectious process resolves, he will be able to tolerate all ß-lactam antibiotics, including ampicillin)

24. A 35-year-old woman with HIV was recently started on trimethoprim-sulfamethoxazole for Pneumocystis carinii prophylaxis. She now presents with fever, sore throat, malaise, and a desquamating rash on her trunk. Laboratory studies are notable for the following abnormalities: serum creatinine, 2.1 mg/dl; aspartate transaminase (AST), 215 mg/dl; and alanine transaminase (ALT), 222 mg/dl.

Which of the following statements regarding the care of this patient is true?
A. She may become hypothyroid as a result of the development of autoimmune thyroiditis within 2 months after the initiation of symptoms
B. In the future, she should avoid sulfonylureas, thiazide diuretics, furosemide, and acetazolamide
C. An elevated serum IgE level confirms the diagnosis of hypersensitivity syndrome reaction
D. Her first-degree relatives have the same risk of experiencing a hypersensitivity syndrome reaction as the general population

Key Concept/Objective: To understand the basic pathophysiology, epidemiology, and clinical manifestations of hypersensitivity drug reactions

Sulfonamide antibiotics can cause hypersensitivity syndrome reactions in susceptible persons. This kind of adverse drug reaction is caused by the accumulation of toxic metabolites; it is not the result of an IgE-mediated reaction. The primary metabolic pathway for sulfonamides involves acetylation of the drug to a nontoxic metabolite and renal excretion. An alternative metabolic pathway, quantitatively more important in patients who are slow acetylators, engages the cytochrome P-450 mixed-function oxidase system. These enzymes transform the parent compound to reactive metabolites—namely, hydroxylamines and nitroso compounds, which produce cytotoxicity independently of preformed drug-specific antibody. In most people, detoxification of the metabolite occurs. However, hypersensitivity syndrome reactions may occur in patients who are unable to detoxify this metabolite (e.g., those who are glutathione deficient). Other aromatic amines, such as procainamide, dapsone, and acebutolol, are also metabolized to chemically reactive compounds. The risk of first-degree relatives’ developing hypersensitivity reactions to sulfonamides is higher than in the general population. Cross-reactivity should not occur between sulfonamides and drugs that are not aromatic amines (e.g., sulfonylureas, thiazide diuretics, furosemide, and acetazolamide); therefore, these drugs can be safely used in the future. Most systemic manifestations of the hypersensitivity reaction syndrome occur at the time of skin manifestations. However, a subgroup of patients may become hypothyroid as part of an autoimmune thyroiditis up to 2 months after the initiation of symptoms. (Answer: A—She may become hypothyroid as a result of the development of autoimmune thyroiditis within 2 months after the initiation of symptoms)

25. A 19-year-old female college student is taking ampicillin and clavulanate for pharyngitis. After 5 days of treatment, she develops a generalized erythematous maculopapular rash. She is given a monospot test, and the result is positive.

For this patient, which of the following statements is true?

A. Exanthematous rashes may occur in up to 80% of patients with infectious mononucleosis that is treated with ampicillin
B. The patient should undergo skin testing with penicilloyl polylysine and graded desensitization before any treatment with penicillins
C. Treatment should include changing to a macrolide antibiotic
D. The patient is experiencing a type II, or cytotoxic, hypersensitivity reaction
E. The rash will worsen until ampicillin is stopped

Key Concept/Objective: To be able to recognize typical ampicillin rash

The causal mechanism of an exanthematous ampicillin rash in the setting of a concurrent viral illness is unclear. It does not appear to be mediated by IgE, so ß-lactams can be tolerated and sensitivity testing is not warranted. Although stopping ampicillin is suggested, the rash will generally resolve even if ampicillin is continued. (Answer: A—Exanthematous rashes may occur in up to 80% of patients with infectious mononucleosis that is treated with ampicillin)

26. In contrast to exanthematous rashes, which of the following is true of urticaria that develops after drug exposure?
A. Type I immediate hyersensitivity reactions cause all urticarial rashes
B. In severe reactions with angioedema and bronchospasm, plasmapheresis should be initiated early in treatment
C. Urticarial rashes remain fixed for up to several days and may recur in the same location with repeated exposure to the causative drug
D. Because of the risk of severe reactions, patients with drug-induced urticaria should not undergo skin testing or desensitization
E. Biopsy should be considered for urticarial lesions that persist for longer than 24 hours

Key Concept/Objective: To know the complications associated with urticarial rashes

For lesions that persist for longer than 24 hours, consideration should be given to the use of biopsy to exclude vasculitis. Biopsy may show deposits of IgM and C3 immune complexes within the lesions. Besides being associated with type I reactions, urticaria may occur with type III hypersensitivity reactions and as a result of nonimmunologic release of histamine caused by certain drugs, such as morphine. Treatment of severe allergic reactions includes epinephrine, antihistamines, bronchodilators, corticosteroids, and supportive treatment with fluids and pressors if needed. Patients can be desensitized if there is no therapeutic alternative to the causative drug. (Answer: E—Biopsy should be considered for urticarial lesions that persist for longer than 24 hours)

27. A 55-year-old woman has a well-demarcated reddish brown macular rash on her arm. The lesion recurs periodically and resolves slowly, with some persisting hyperpigmentation. She is otherwise healthy and takes no medications except an occasional laxative.

Which of the following is the most likely diagnosis for this patient?
A. Urticaria
B. Lichen planus
C. Pemphigus
D. Fixed drug eruption
E. Contact dermatitis

Key Concept/Objective: To be able to diagnose fixed drug eruption in the appropriate setting

Fixed drug eruptions may occur after ingestion of several over-the-counter medications, including phenolphthalein laxatives and ibuprofen. After an exacerbation, a refractory period may occur during which reexposure does not produce a recurrence of the rash, so the diagnosis may be elusive. The most common site of involvement is the genitalia, so fixed drug eruptions must be distinguished from various sexually transmittable afflictions. (Answer: D—Fixed drug eruption)

28. Which of the following is true concerning the development of cutaneous necrosis in a patient taking warfarin?
A. Skin lesions appear weeks to months after beginning treatment
B. The pretibial area is the most common site
C. Lesions generally occur only when the INR exceeds 3.5
D. Treatment includes heparinization
E. Patients with lupus anticoagulant or antithrombin III deficiency are predisposed

Key Concept/Objective: To understand the pathogenesis and treatment of warfarin-induced skin necrosis

Patients with protein C or protein S deficiency may be predisposed to develop warfarininduced skin necrosis. They develop a paradoxical hypercoagulable state at the onset of treatment because of suppression of protein C anticoagulant activity, resulting in venous thrombosis and necrosis 3 to 5 days later. Fatty areas are most frequently affected. Heparin, vitamin K, and fresh frozen plasma are the mainstays of treatment. (Answer: D—Treatment includes heparinization)

29. One month after starting phenytoin after a head injury, a 24-year-old man developed a low-grade fever, cervical lymphadenopathy, and a generalized erythematous maculopapular rash with subsequent exfoliation in some areas.

Which of the following statements is true of this condition?
A. It is commonly associated with penicillins and ACE inhibitors
B. Limited laboratory investigation consisting of a complete blood count and a urinalysis is warranted
C. Graves disease is a late complication
D. First-degree relatives of the patient are at increased risk for similar reactions
E. After the cutaneous reactions, rechallenge and desensitization are advised before reinstituting therapy

Key Concept/Objective: To be able to recognize the signs and complications of hypersensitivity syndrome

Hypersensitivity syndrome is a potentially serious reaction occurring from 1 week to several weeks after exposure to aromatic anticonvulsants (e.g., phenytoin, carbamazepine), sulfonamides, or other drugs with an aromatic amine chemical structure (procainamide). Inheritable defects in the metabolic pathways for these agents may place close relatives at increased risk as well. Eosinophilia, hepatitis, and interstitial nephritis may be detected initially, and autoimmune thyroiditis can cause late hypothyroidism. The rash may range from an exanthem to severe Stevens-Johnson syndrome or toxic epidermal necrolysis. Because these reactions are severe (and not IgE-mediated), patients and family members are advised to avoid the causative drug and drugs that are chemically similar to it. (Answer: D—First-degree relatives of the patient are at increased risk for similar reactions)

For more information, see Shear NH, Knowles S, Shapiro L: 2 Dermatology: VI Cutaneous Adverse Reactions. ACP Medicine Online (www.acpmedicine.com). Dale DC, Federman DD, Eds. WebMD Inc., New York, March 2004

Fungal, Bacterial, and Viral Infections of the Skin

30. A 6-year-old boy comes with his mother to your clinic with a scalp lesion. He developed this lesion a few weeks ago. On physical examination, the patient has an area of alopecia on his scalp; associated with the alopecia is a painful inflammatory mass with pus and sinus tracts. A skin specimen treated with potassium hydroxide (KOH) shows the presence of dermatophytes. A Gram stain shows no bacterial organisms.

What is the likely diagnosis for this patient, what is the causal organism, and how should his condition be treated?
A. Kerion; Microsporum or Trycophyton; oral griseofulvin
B. Bacterial abscess; Staphylococcus aureus; oral dicloxacillin
C. Fungal and bacterial coinfection; Trychophyton and Staphylococcus aureus; oral itraconazole and dicloxacillin
D. Sebaceous tumor; surgical removal

Key Concept/Objective: To understand the clinical picture and treatment of kerion

Tinea capitis occurs primarily in children. Only Microsporum and Trychophyton species can cause tinea capitis. Zoophilic species, such as Microsporum or Trycophyton, can provoke intense inflammation, which can result in a kerion—a painful, boggy mass in which follicles may discharge pus and in which sinus tracts form. Crusting and matting of adjacent hairs are common, and cervical lymph nodes may enlarge. The diagnosis is confirmed by obtaining a sample from the lesion, treating it with KOH (which digests the keratin of the skin and hair), and examining it under the microscope for the presence of organisms. Although a bacterial infection or coinfection needs to be considered in the differential diagnosis, the presence of pus in this case is related to the intense inflammatory reaction against the fungus and does not necessarily mean that the patient has a bacterial infection. Oral therapy is necessary when treating fungal infections involving the hair or hair follicles or in extensive lesions. The agents of choice for the treatment of tinea capitis are griseofulvin and terbinafine. (Answer: A—Kerion; Microsporum or Trycophyton; oral griseofulvin)

31. A 26-year-old white man presents to a walk-in clinic complaining of a rash on his back. He noticed the rash 4 or 5 weeks ago. He describes small, whitish lesions that are not painful and do not itch on his back. On physical examination, the patient is seen to have several small, dark macules that coalesce on his upper back. When the lesions are scratched, fine scales are produced.

How should you proceed in the management of this patient?

A. A fungal culture from the lesion should be obtained
B. A skin biopsy should be performed
C. The lesions should be scraped and a KOH stain should be performed
D. The patient should be started on oral terbinafine

Key Concept/Objective: To know the clinical picture of and diagnostic approach to tinea versicolor

This patient’s presentation is consistent with tinea versicolor. Tinea (or pityriasis) versicolor is a yeast infection caused by Malassezia furfur. The lesions are small, discrete macules that tend to be darker than the surrounding skin in light-skinned patients and hypopigmented in patients with dark skin. They often coalesce to form large patches of various colors ranging from white to tan. Scratching the lesions produces a fine scale. This infection most commonly involves the upper trunk, but the arms, axillae, abdomen, and groin may also be affected. To confirm the diagnosis, a KOH preparation of scrapings from the lesions can be done, which can demonstrate pseudohyphae and yeasts resembling spaghetti and meatballs. Because these yeasts form part of the normal cutaneous flora, growth of the organism on cultures from the skin is not very helpful diagnostically. Treatment of tinea versicolor involves applying selenium sulfide shampoo topically. An alternative is the use of topical azoles such as ketoconazole, miconazole, and clotrimazole. For patients who have difficulty using topical agents, oral ketoconazole or fluconazole is an alternative. Terbinafine is not active against yeasts. (Answer: C—The lesions should be scraped and a KOH stain should be performed)

32. A 34-year-old man comes to your office complaining of a skin ulcer. He first noticed a skin lesion 6 or 7 days ago. It started as a small, painless papule on his right arm. Over the next few hours, the lesion enlarged, and the patient noticed significant swelling around the lesion. After a few days, he developed a black eschar, which sloughed the day before the visit, leaving a painless ulcer. On physical examination, the patient has a painless ulcer measuring 2 × 2 cm that is surrounded by significant edema and that has a tender, epitrochlear node. A Gram stain of the ulcer shows broad gram-positive rods. The patient is not allergic to penicillin. You have heard of similar cases in a local hospital.

What is the next step in the treatment of this patient?
A. Start penicillin V, 500 mg p.o., q.i.d., for 60 days and send cultures and serology to confirm your clinical diagnosis; change antibiotics according to the culture results
B. Start ciprofloxacin, 500 mg p.o., q.d., for 10 days and send cultures and serology to confirm your clinical diagnosis; change antibiotics according to the culture results
C. Start amoxicillin, 500 mg p.o., t.i.d., for 10 days and send cultures and serology to confirm your clinical diagnosis; change antibiotics according to the culture results
D. Start ciprofloxacin, 500 mg p.o., q.i.d., for 60 days and send cultures and serology to confirm your clinical diagnosis; change antibiotics according to the culture results

Key Concept/Objective: To know the appropriate treatment of cutaneous anthrax

Anthrax was very rare in the United States until 2002, when spores of Bacillus anthracis were sent through the mail as an act of terrorism. Except for cases associated with bioterrorism, humans usually develop anthrax from exposure to affected animals or their products. Occasionally, laboratory-acquired cases can occur. The cutaneous form develops when spores enter the skin through abrasions and then transform into bacilli, which produce edema and necrosis. After an incubation period of about 1 to 7 days, a painless, pruritic papule forms at the entry site. Over the next few hours, the lesion enlarges and a ring of erythema appears. Painless edema surrounds the lesion, often spreading to the adjacent skin and soft tissue. In the center of the lesion, a black eschar appears and sloughs within 1 to 2 weeks, leaving a shallow ulcer that heals with minimal scarring. Regional lymph nodes often enlarge, causing pain and tenderness. B. anthracis is a broad, encapsulated gram-positive rod, which can be seen on a Gram stain of material from the skin lesion. It grows readily on blood agar media. Skin biopsies reveal necrosis, hemorrhage, and edema. Organisms are demonstrable with tissue Gram stain or immunohistochemical staining. Because serologic testing requires acute and convalescent blood specimens, such testing is unhelpful for immediate diagnosis but may establish a retrospective diagnosis of a suspected but unconfirmed case. The treatment for cutaneous anthrax that is not associated with bioterrorism is a regimen of penicillin V or amoxicillin for 7 to 10 days. In this patient, there is a chance that anthrax is related to a bioterrorist attack because similar cases have been seen in the past few days; therefore, ciprofloxacin should be started. The recommended regimen for cases associated with bioterrorism is 60 days because of the possibility of simultaneous aerosol exposure. (Answer: D—Start ciprofloxacin, 500 mg p.o., q.i.d., for
60 days and send cultures and serology to confirm your clinical diagnosis; change antibiotics according to the culture results)

For more information, see Hirschmann JV: 2 Dermatology: VII Fungal, Bacterial, and Viral Infections of the Skin. ACP Medicine Online (www.acpmedicine.com). Dale DC, Federman DD, Eds. WebMD Inc., New York, March 2003

Parasitic Infestations

33. A 22-year-old graduate student recently returned from a trip to central Mexico. She subsequently developed several nontender nodules on her lower back that intermittently drain brown fluid. She received treatment with dicloxacillin, without improvement. On examination, she has five purple, firm nodules that measure approximately 2 cm; the nodules have tiny central openings that intermittently drain serosanguineous fluid.

What would you do next for this patient?
A. Give her a 10-day course of oral cephalexin
B. Give her a 10-day course of oral levofloxacin
C. Excise a nodule for pathologic evaluation
D. Give her a 10-day course of I.V. gentamicin
E. Apply fatty bacon over the nodules

Key Concept/Objective: To be able to identify and manage patients with myiasis

This patient has just returned from an area endemic for the botfly. The nodules with a central punctum that intermittently drain are a characteristic of infestation with larvae of that insect. Not infrequently, patients with myiasis are misdiagnosed as having bacterial furunculosis and are managed with a variety of antibiotics that have no effect. It is important to consider the diagnosis of myiasis because occluding the punctum with fatty bacon causes the larvae to protrude. The larvae can then be removed with forceps. Other occlusive substances, such as petroleum jelly and nail polish, have also been effective. Excising the nodule for pathologic evaluation is not needed. (Answer: E—Apply fatty bacon over the nodules)

34. A 37-year-old woman presents with intense perineal itching. On examination, she has both red papules and blue macules on the inner thighs. In addition, there are excoriated, crusted lesions in the same region. Tiny tan swellings are seen at the bases of some of the pubic hair shafts.

What is your diagnosis?
A. Dermatitis herpetiformis
B. Genital herpesvirus
C. Pediculosis
D. Scabies
E. Contact dermatitis

Key Concept/Objective: To know the distinguishing characteristics of diseases that cause pruritic skin lesions

This patient has pediculosis pubis, caused by Phthirus pubis. Its characteristic blue-gray macules (maculae ceruleae) are caused by the pubic lice sucking blood from the dermis. The tan swellings are nits: lice eggs cemented to the hair shaft. None of the other diseases listed have these findings. Scabies is caused by a burrowing mite that can at times be seen as a line in the stratum corneum. The mite’s eggs are deposited in the burrows rather than as nits. Dermatitis herpetiformis is an autoimmune vesicular dermatitis characterized by grouped vesicles symmetrically distributed over the extensor skin surfaces. Genital herpes can be pruritic but is more often painful. Contact dermatitis can cause pruritic lesions but is not caused by a parasitic infestation. (Answer: C—Pediculosis)

35. A 32-year-old man returns from a vacation to the Caribbean. He spent time swimming and sunbathing at the local beaches. He now has intensely pruritic skin lesions on the abdomen. Examination reveals several serpiginous thin lines in the skin.

Which of the following organisms is most likely responsible for this disease?

A. Botfly larvae
B. Avian schistosome free-swimming larvae (cercariae)
C. Hookworm larvae
D. Thimble jellyfish larvae
E. Sandfleas

Key Concept/Objective: To understand the causes and presentations of skin lesions that can occur during travel to tropical regions

Tungiasis (sandflea cutaneous infestation), seabather’s eruption (infestation with larvae of the thimble jellyfish), swimmer’s itch (infestation with cercarial larvae of an avian schistosome), and cutaneous larva migrans (infestation with hookworm larvae) can all cause intensely pruritic skin lesions. It is the migration of larval hookworms that causes the serpiginous or linear tracks in the skin. Tungiasis causes erythematous edematous papules in clusters. Seabather’s eruption presents as a pruritic dermatitis in areas covered by swimwear. Swimmer’s itch is a papulovesicular eruption on exposed skin sites. The larvae of the botfly cause myiasis, a nonpruritic nodular skin lesion mimicking an infected cyst or abscess. (Answer: C—Hookworm larvae)

36. Two months after returning from an expedition to the Amazon, a 42-year-old archeologist notices a redbrown papule on her nose. On skin examination, she is suntanned. She also has a verrucous nodule with early ulceration where she first noted the papule on the nose.

What would you do for this patient at this point?

A. Perform biopsy of the nodule for histopathology
B. Treat with oral sodium stibogluconate (a pentavalent antimony compound)
C. Ablate with cryosurgery
D. Inject the nodule intralesionally with antimonials
E. Treat with oral ivermectin

Key Concept/Objective: To understand the presentation of cutaneous leishmaniasis and its management

This patient’s clinical examination and course fit the diagnosis of New World cutaneous leishmaniasis. The differential diagnosis includes various inflammatory and neoplastic disorders, including squamous cell carcinoma. The safest course in this case would be to perform biopsy of the lesion to confirm the diagnosis. Once the diagnosis is confirmed, the patient could be treated with oral sodium stibogluconate. Old World leishmaniasis is usually limited to the skin and can be treated with cryosurgery, heat therapy, or intralesional injection of antimonials. Ivermectin is therapy for scabies, pediculosis, and cutaneous larva migrans. (Answer: A—Perform biopsy of the nodule for histopathology)

37. A 17-year-old boy living at home with his parents presents with an intensely pruritic papulovesicular eruption involving the hands and wrists. Skin scrapings identify eggs and waste products of Sarcoptes scabiei.

Which of the following management options is most appropriate?
A. Treat the patient and symptomatic household members with permethrin 5% cream, and tell them to wash all clothing and linens with which they have come in contact over the past 2 days
B. Treat the patient and all household members with permethrin 5% cream, and tell them to wash all clothing and linens with which they have come in contact over the past 2 days
C. Treat the patient and symptomatic household members with permethrin 5% cream, and tell them to wash all clothing and linens with which they have come in contact over the past 10 days
D. Treat the patient and all household members with permethrin 5% cream, and tell them to wash all clothing and linens with which they have come in contact over the past 10 days
E. Treat the patient and symptomatic household members with lindane lotion, and tell them to wash all clothing and linens with which they have come in contact over the past 10 days

Key Concept/Objective: To understand the treatment of scabies

Both permethrin 5% cream and lindane are effective therapies for scabies. However, treatment involves more than just choosing a scabicidal agent. Because the skin flakes that are shed by patients contain large numbers of mites, fomites can spread. The recommendation is to treat all members of the household, not just those who are symptomatic. The organism’s ability to survive for 48 hours away from a host dictates thorough laundering of materials that might have been contaminated within the previous 2 days. (Answer: B—Treat the patient and all household members with permethrin 5% cream, and tell them to wash all clothing and linens with which they have come in contact over the past 2 days)

For more information, see Abel EA: 2 Dermatology: VIII Parasitic Infestations. ACP Medicine Online (www.acpmedicine.com). Dale DC, Federman DD, Eds. WebMD Inc., New York, March 2001

Vesiculobullous Diseases

38. A 45-year-old man comes to your clinic complaining of skin and mouth lesions. Painful oral ulcers started to appear in his mouth 1 month ago. Over the past week, he has also noticed some skin lesions on his upper chest and back. He has no significant medical history and is not taking any medications. His physical examination is unremarkable except for the presence of several superficial tender ulcers on his oral mucosa and six superficial coin-sized lesions on his back and chest, which are surrounded by normal skin. The Nikolsky sign is positive.

On the basis of clinical presentation, which of the following is the most likely diagnosis for this patient?
A. Bullous pemphigoid
B. Pemphigus vulgaris
C. Pemphigus foliaceus
D. Porphyria cutanea tarda

Key Concept/Objective: To know the typical presentation of pemphigus vulgaris

Pemphigus is characterized by blisters that arise within the epidermis and by a loss of cohesion of the epidermal cells. Pemphigus vulgaris is the most common form of pemphigus. It can develop at any age but usually occurs in persons between 30 and 60 years old. Pemphigus vulgaris usually begins with chronic, painful, nonhealing ulcerations in the oral cavity. Skin lesions can also be the initial manifestation, beginning as small fluidfilled bullae on otherwise normal-looking skin. The blisters are usually flaccid because the overlying epidermis cannot sustain much pressure. Bullae therefore rupture rapidly, usually in several days, and may be absent when the patient is examined. Sharply outlined, coin-sized, superficial erosions with a collarette of loose epidermis around the periphery of the erosions may appear instead. The upper chest, back, scalp, and face are common sites of involvement. A characteristic feature of all active forms of pemphigus is the Nikolsky sign, in which sliding firm pressure on normal-appearing skin causes the epidermis to separate from the dermis. Pemphigus foliaceus usually begins with crusted, pruritic lesions resembling corn flakes on the upper torso and face; oral involvement is very rare. Bullous pemphigus is characterized by recurrent crops of large, tense blisters arising from urticarial bases. Porphyria cutanea tarda appears as blistering lesions in sun-exposed areas, typically on the dorsa of hands. (Answer: B—Pemphigus vulgaris)

39. A 40-year-old man comes to clinic complaining of skin lesions. His symptoms started 6 weeks ago, with crusted lesions localized on his face, back, and chest; he denies having any oral lesions. He has no medical history and is not taking any medications. On physical examination, the patient has several crusted lesions on his face, upper chest, and back. Results of a skin biopsy are consistent with pemphigus foliaceus.

Of the following, which is the most appropriate treatment for this patient?
A. Plasmapheresis
B. Intravenous immunoglobulin
C. Cyclophosphamide
D. Prednisone

Key Concept/Objective: To understand the treatment of pemphigus foliaceus

Initial therapy for pemphigus is determined by the extent and rate of progression of lesions. Localized, slowly progressive disease can be treated with intralesional injections of corticosteroids or topical application of high-potency corticosteroids. New lesions that continue to appear in increasing numbers can be controlled in some cases with low-dose systemic corticosteroids (prednisone, 20 mg/day). Patients with extensive or rapidly progressive disease are treated with moderately high doses of corticosteroids. If disease activity persists despite high doses of corticosteroids, one of the following approaches should be considered for rapid control: plasmapheresis; intravenous immunoglobulin; or pulse therapy with high-dose intravenous methylprednisolone. Although pemphigus foliaceus is less severe than pemphigus vulgaris, the doses of medications required for treatment of both diseases are similar. (Answer: D—Prednisone)

40. A 30-year-old woman comes to clinic complaining of blisters on her body. She started developing these lesions a week ago. The lesions consist of very pruritic, small blisters on her arms, buttocks, and back. She says she had a similar lesion a few months ago that resolved on its own after a few weeks. Physical examination is significant for multiple small papules and vesicles on the elbows, buttocks, and lower back; there are signs of previous scratching. A skin biopsy specimen shows microabscesses at the tips of dermal papillae and granular deposits of IgA on the basement membrane zone.

On the basis of this patient’s clinical presentation, what is the most likely diagnosis, and what treatment would you prescribe?
A. Erythema multiforme; prednisone
B. Pemphigus vulgaris; prednisone
C. Dermatitis herpetiformis; dapsone
D. Pemphigus foliaceus; azathioprine

Key Concept/Objective: To know the clinical presentation and management of dermatitis herpetiformis

Dermatitis herpetiformis is a vesiculobullous disease characterized by intensely pruritic, small vesicles that are grouped in small clusters and typically appear on the extensor aspects of extremities and on the buttocks, scalp, and back. The condition is believed to be an immune-mediated disorder and is associated with abnormal granular deposits of IgA at the basement membrane zone and with asymptomatic, gluten-sensitive, spruelike enteropathy. The disease is chronic, with periods of exacerbation and remission. Lesions may clear if patients follow a strict gluten-free diet. Dermatitis herpetiformis responds rapidly and dramatically to dapsone. Erythema multiforme is characterized by the presence of target lesions; it commonly affects mucosal surfaces. The histologic findings include subepidermal edema and a deep perivascular mononuclear infiltrate, sometimes with granular deposits of C3 or IgM. Pemphigus vulgaris and foliaceus show acantholysis; on immunofluorescence, intercellular autoantibodies, IgG, IgM, or IgA is seen. (Answer: C— Dermatitis herpetiformis; dapsone)

For more information, see Abel EA, Bystern JC: 2 Dermatology: IX Vesiculobullous Diseases. ACP Medicine Online (www.acpmedicine.com). Dale DC, Federman DD, Eds. WebMD Inc., New York, July 2003

Malignant Cutaneous Tumors

41. A 64-year-old retired Navy officer presents to clinic for a routine health maintenance visit. He has no complaints, but when asked about a pinkish papular lesion near the corner of his left eye, he states that it has been present “for years” and that it has become irritated on occasion with minor trauma or rubbing. The lesion is 4 to 5 mm in diameter and appears pearly. You recommend that the patient undergo biopsy because you are concerned about the possibility of basal cell carcinoma (BCC).

Which of the following epidemiologic and clinical statements is NOT true of BCC?
A. BCC is the most common skin cancer
B. The vast majority of BCCs occur on the head and neck
C. Based on appearance, this patient’s lesion is likely a nodular BCC
D. Metastatic disease and deaths associated with BCC have been reported
E. BCC arises from cells in the dermis

Key Concept/Objective: To understand fundamental aspects of basal cell carcinoma, a form of nonmelanoma skin cancer

Malignant tumors can arise from cells of any layer of the skin, and epidermal skin cancers are the most common cancers in humans. Basal cell carcinoma (BCC) and squamous cell carcinoma (SCC) arise from keratinocytes of the epidermis, whereas malignant melanoma arises from the melanocytes of the epidermis. Because BCC and SCC share many features, they are often lumped together under the term nonmelanoma skin cancer. BCC is the most common skin cancer and is more prevalent in people with lighter skin pigmentation. Over 90% of BCC lesions appear on the head and neck. Sun exposure is the most important risk factor. The two most common forms are nodular BCC (which is likely the type that this patient demonstrates) and superficial BCC, which appears as a pink patch of skin. Although rare, metastases and death from BCC have been known to occur, and any suspicious lesions (especially long-standing ones in a sun-exposed area that easily bleed with minor trauma) should be excised and submitted for pathologic examination. (Answer: E— BCC arises from cells in the dermis)

42. A 50-year-old construction worker with hypertension presents for a return office visit. His blood pressure appears well controlled, and you are discussing other health maintenance issues as you examine him. You notice several rough-surfaced, irregularly shaped lesions on his face, scalp, and dorsa of the hands. On closer inspection, they appear hyperkeratotic and have a small rim of surrounding erythema. He says they are not painful, do not itch, and have been appearing over the course of years.

Which of the following statements regarding this patient’s risk of skin cancer is true

A. The lesions are precursors to melanoma and should be removed
B. This patient’s risk of developing a cutaneous malignancy in relation to the lesions is less than 2%, and he should be reassured that they are completely benign
C. Treatment of the lesions by methods such as cryotherapy, curettage, or topical chemotherapy has been found to be effective in preventing the progression of such lesions to carcinoma
D. Small squamous cell carcinomas arising in the areas described are more likely to metastasize than are more undifferentiated lesions developing in non–sun-exposed areas
E. The most important risk factor in the development of these lesions is family history

Key Concept/Objective: To know that actinic keratosis is a potential precursor to squamous cell carcinoma of the skin

This patient has hyperkeratotic lesions typical of actinic keratosis in sun-exposed areas. Actinic keratosis is seen in areas of chronically sun-damaged skin and is considered a precursor lesion to the development of SCC. The majority of patients with actinic keratosis have multiple lesions, and the risk of SCC in these patients is estimated to be as high as 20%. Thus, it is important that the patient be followed regularly and evaluated by a dermatologist: the removal of these lesions through various techniques can prevent progression to cancer. Small SCCs that arise from actinic keratosis lesions are actually less likely to metastasize than more atypical SCCs, such as those that are poorly differentiated or appear in non–sun-exposed areas or oral or genital mucosa. Sunlight exposure is the most important risk factor for developing actinic keratosis and SCC, although radiation, chemical burns, and chronic nonhealing wounds may also predispose to squamous cell cancer. (Answer: C—Treatment of the lesions by methods such as cryotherapy, curettage, or topical chemotherapy has been found to be effective in preventing the progression of such lesions to carcinoma)

43. A 59-year-old white woman with rheumatoid arthritis who was treated in the past with methotrexate and courses of steroids presents for evaluation of a mole on her chest. She states that it has been present for years but that, in the past 6 to 8 months, she noticed more irregularity at the borders and an increase in the size of the lesion. Examination reveals an asymmetrical lesion approximately 8 mm in diameter that is variably pigmented from brown to black. You recommend biopsy of the lesion because you are concerned about malignant melanoma.

If a primary cutaneous melanoma is confirmed, which of the following factors would be the most important with regard to outcome in this patient?
A. Evolution of the lesion from a dysplastic nevus
B. History of immunosuppressive therapies
C. Tumor diameter greater than 6 mm
D. Tumor thickness
E. Location of the melanoma

Key Concept/Objective: To understand the importance of tumor thickness as a prognostic factor in primary cutaneous melanoma

Malignant melanoma is the most aggressive of the primary cutaneous malignancies, and the clinician should have a high index of suspicion when evaluating moles with the characteristics of melanoma. The “ABCD” mnemonic is useful for remembering the features of melanoma: asymmetry, border irregularity, color variation, and diameter greater than 6 mm. Such features warrant biopsy of the lesion. In this patient, the change in the size of a mole over time also warrants prompt evaluation. The single strongest prognostic factor in melanoma is stage of disease at the time of diagnosis. Staging takes into account tumor size, nodal involvement, and distant metastases. For primary tumors, the most consistent factor predictive of outcome is tumor thickness, as described by the Breslow depth. Patients with dysplastic nevi are at increased risk for the development of melanomas; the presence of dysplastic nevi does not consistently relate to the prognosis of patients with melanoma. (Answer: D—Tumor thickness)

44. A 35-year-old white man presents at a walk-in clinic with a complaint of lesions in his mouth and over his trunk. These lesions developed over the past several months. His medical history is unremarkable. He states that he is homosexual, that he has practiced unsafe sex in the past, and that he has had the same partner for the past 18 months. He denies having previously had any sexually transmitted diseases, but he says he has not had regular health care visits since high school. On examination, you note numerous purple-red, oval papules distributed on the trunk and two deep-purple plaques on the soft palate and buccal mucosa. The patient also has several small, firm, nontender, palpable lymph nodes in the posterior cervical, axillary, and inguinal chains. Results of routine blood work are unremarkable except for a white blood cell count of 3,000 cells/mm3 and a differential with 5% lymphocytes.

Which of the following statements regarding our current knowledge of Kaposi sarcoma (KS) is false?
A. Human herpesvirus 8 (HHV-8) plays an etiologic role exclusively in
HIV-associated KS
B. Visceral organ (i.e., lung and GI tract) involvement is relatively common in HIV-associated KS
C. If HIV infection is confirmed, initiation of highly active antiretroviral therapy (HAART) in this patient would likely lead to dramatic improvements in the lesions during the first few months of therapy
D. Male sex is a significant risk factor for the condition, especially in the classic form of the disease
E. Total CD4+ T cell count is the most important factor predictive of survival in the form of this disease associated with HIV

Key Concept/Objective: To be able to recognize KS and appreciate important aspects of its diagnosis and treatment

This patient is a homosexual man who presents with skin and oral lesions typical of KS. The additional findings of generalized lymphadenopathy and lymphopenia strongly suggest that the patient is infected with HIV. In its classic form, KS affects elderly men, primarily of Mediterranean descent, and manifests as violaceous plaques and nodules on the lower extremities. The disease was rare in the United States before the AIDS epidemic. Among HIV-infected patients, homosexual men have by far the highest incidence of KS. Recently, it has been shown that HHV-8 can be detected in all variants of KS, suggesting an etiologic role. HIV-associated KS presents as oral lesions or cutaneous lesions on the upper body. They often follow the skin lines in a pityriasis rosea-like distribution. KS can involve the pulmonary and gastrointestinal systems and can cause hemorrhage at these sites. As such, a chest x-ray and fecal occult blood test should be considered when evaluating patients newly diagnosed with HIV-associated KS. The single most important prognostic factor in HIV-associated KS is the CD4+ T cell count. Large tumor burdens, lymphedema, and pulmonary involvement also portend a poorer outcome. HAART is often first-line therapy in treating KS, especially in a patient with newly diagnosed AIDS. The improvement in viral load and CD4+ T cell counts is often accompanied by regression of KS lesions. Other therapeutic options include radiation, intralesional chemotherapy injections, and systemic chemotherapy, including liposomally encapsulated anthracyclines such as doxorubicin and daunorubicin. (Answer: A—Human herpesvirus 8 [HHV-8] plays an etiologic role exclusively in HIV-associated KS)

45. A middle-aged woman comes to clinic complaining of a long-standing rash involving her chest and left thigh, which she first noted over a year ago. She says the areas are chronically red and scaly and are occasionally mildly pruritic. She has not been able to identify any precipitating factors or irritants that have come into contact with those particular areas. She states that exposure to the sun has intermittently made the lesions improve to an extent. Approximately 6 months ago, she was prescribed a topical steroid cream, which did seem to cause improvement of the rash, but the rash soon returned after discontinuation. She thought that the rash likely represented psoriasis, and she had not been overly concerned about it until recently, when she has noticed that the lesions had become larger and more prominent. On examination, you note a large erythematous, scaly patch on the trunk. The lesion on the upper thigh is a thicker plaque that is deeper red in color. Skin biopsy reveals atypical lymphoid cells in the epidermis that have hyperconvoluted (cerebriform) nuclei. There is also a bandlike lymphocytic infiltrate in the upper dermis. A diagnosis of mycosis fungoides is made on the basis of the histologic report.

Which of the following clinical and therapeutic statements is NOT characteristic of this disorder?
A. The rash is caused by a cutaneous lymphoma that is most commonly of B cell origin
B. The condition can be associated with generalized erythroderma and circulating atypical lymphocytes with cerebriform nuclei
C. Early-stage disease that is confined to patches or plaques in the skin is primarily treated with topical therapy involving steroids or chemotherapeutic agents
D. Ulcerations from the cutaneous lesions should be monitored closely and treated aggressively because sepsis originating from these lesions is a common cause of death in these patients
E. Patients with visceral involvement have a poor prognosis; the median survival is 2.5 years

Key Concept/Objective: To be able to recognize the clinical manifestations of cutaneous T cell lymphoma (CTCL)

Non-Hodgkin lymphomas may primarily involve the skin and present as chronic, erythematous patches or plaques. The vast majority of these are T cell in origin and are known as cutaneous T cell lymphomas (CTCLs). Mycosis fungoides is the name commonly applied to this condition, although there are other variants. Five percent of patients with CTCL present with Sézary syndrome and have generalized erythroderma and circulating atypical T cells (Sézary cells); this condition represents the leukemic variant of CTCL. CTCL can be difficult to diagnose, given its indolent course and the fact that its appearance is similar to those of other benign inflammatory conditions of the skin, including psoriasis, eczema, contact dermatitis, and drug reactions. The lesions typically appear in a bathing trunk distribution. Staging of the disease is based on the surface area of skin involved with patches or plaques and the involvement of lymph nodes, visceral organs, and blood. Earlystage disease is primarily treated with topical therapy (such as corticosteroids, nitrogen mustard, or carmustine) and radiation of the lesions. Advanced disease is associated with a poor prognosis. The most serious complications of CTCL are infections: sepsis from ulcerated cutaneous tumors is a common cause of death in these patients. (Answer: A—The rash is caused by a cutaneous lymphoma that is most commonly of B cell origin)

For more information, see Halpern AC: 2 Dermatology: X Malignant Cutaneous Tumors. ACP Medicine Online (www.acpmedicine.com). Dale DC, Federman DD, Eds. WebMD Inc., New York, October 2002

Benign Cutaneous Tumors

46. A 56-year-old white man presents to your office for evaluation of bumps on his upper back and chest.
His wife reports that he has had them for years, but they seem to be increasing in number. He states that they do not itch or hurt. Examination reveals several sharply circumscribed papules measuring from 2 mm to 2 cm in diameter on the patient’s upper back and chest. The lesions are light brown and have a stuck-on appearance. Closer examination reveals follicular plugging.

What is the most likely diagnosis for this patient?
A. Nevus cell nevus
B. Pigmented basal cell carcinoma
C. Seborrheic keratosis
D. Dermatosis papulosa nigra

Key Concept/Objective: To be able to recognize seborrheic keratosis and to be familiar with the differential diagnosis of lesions with a similar appearance

This patient’s lesions are consistent with seborrheic keratosis (seborrheic wart). Seborrheic keratosis is a very common epithelial tumor that tends to occur on the upper trunks of light-skinned adults. They occur more frequently with increasing age. The color can range from dirty yellow to dark brown, and their size varies from 1 mm to several cm. They may be rough or smooth but often have a waxy surface. Dermatosis papulosa nigra is similar to seborrheic keratosis but tends to occur in dark-skinned individuals (this patient is white) and is usually localized on the face. In addition, dermatosis papulosa nigra tends to present at an earlier age than does seborrheic keratosis. The differential diagnosis of seborrheic keratosis also includes lentigo, warts, nevus cell nevus, and pigmented basal cell carcinoma. Inflamed seborrheic keratosis can be difficult to distinguish from malignant melanoma and squamous cell carcinoma. Transient development of seborrheic keratosis has been associated with inflammatory skin conditions such as drug-related erythroderma and psoriasis. The sign of Leser-Trelat is transient eruptive seborrheic keratosis that is associated with internal malignancy (especially adenocarcinoma); the validity of this sign is a subject of debate. (Answer: C—Seborrheic keratosis)

47. A 35-year-old woman presents with a lump on her back. She is worried about having cancer. The lump has been there for several months and has grown some over time. Otherwise, the patient is in good health and takes no medications. There is no family history of malignancy. Examination is notable for a firm, rubbery nodule measuring 1 cm on her upper back. The nodule seems to be just under the surface of the skin and is not fixed in place. The borders are smooth, there is no abnormal pigmentation, and there is a small pore in the center of the lesion. You tell the patient she most likely has an epidermoid cyst, and you attempt to reassure her.

Which of the following is NOT a treatment option for epidermoid cysts?
A. Systemic antibiotics and warm-water compresses if the cyst becomes infected or inflamed
B. Incision of the cyst with a pointed scalpel, and expression of the cyst wall and its contents
C. Excision of the entire cyst
D. Cryotherapy

Key Concept/Objective: To be able to recognize the presentation of an epidermoid cyst and to be familiar with treatment options for such a lesion

Epidermoid cysts, or wens, are common and appear to be derived from hair follicles. They are frequently found on the back as firm nodules measuring 0.2 to 5.0 cm in diameter. They are slow-growing and often have a central pore. They are asymptomatic unless they become inflamed or infected. In such cases, the patient should receive antibiotics and have warm-water compresses applied three or four times a day. After the inflammation or infection has resolved, the patient can have the cyst removed. Removal in other cases is usually for cosmetic reasons. The cyst may recur if the cyst wall is not removed. Therefore, treatment options include simple incision and expression of the cyst’s contents and wall or, for more fibrotic cysts, surgical excision of the entire cyst. Cryotherapy is not useful for removal of the cyst. Pilar cysts are very similar in appearance to wens but have a semifluid, malodorous core. Milia are smaller and firmer than wens, and they tend to be located on the face and in scars. Treatment is similar for all these cysts. (Answer: D—Cryotherapy)

48. A 56-year-old farmer presents for a routine health examination. He states that he has been doing well. On examination, you note that the patient has some sun damage to his skin and that he has a dark complexion. There is a hyperpigmented, slightly raised lesion measuring 1 cm on his left forearm. There is no lymphadenopathy in his arm or axilla. The patient states that he does not really remember noticing this lesion before; he denies using any sunscreen. You are worried that the lesion on his forearm may be a dysplastic nevus or melanoma.

Which of the following features of this patient’s hyperpigmented lesion would NOT make it more likely to be a dysplastic nevus or melanoma

A. Small size (< 5 mm)
B. Flatness
C. Irregular pigmentation
D. Indistinct borders

Key Concept/Objective: To understand the features of hyperpigmented lesions that make them more likely to be a dysplastic nevus or melanoma than a nevus cell nevus (melanocytic nevus)

Nevus cell nevus (melanocytic nevus) is the most common skin tumor, and most young adults have 20 to 40 of these lesions. The incidence increases with age up to the second or third decade, then declines. These nevi are more common in sun-exposed areas. It is important to realize that the risk of melanoma increases with the number of melanocytic nevi. However, the presence of even one dysplastic nevus increases a person’s risk of melanoma. Therefore, it is important to be familiar with the appearance of dysplastic nevi (the features of which are similar to those of melanoma). Features include large size (> 5 mm), flatness, irregular pigmentation, asymmetry, and indistinct borders. (Answer: A— Small size [< 5 mm])

49. A woman brings her 13-year-old son to your clinic for evaluation of multiple lumps and bumps. She states that her husband had similar problems and died of a nervous system disease. She does not remember the name of her husband’s disease, but notes that his tumors were at times large and painful and that on numerous occasions he had to have some surgically removed. The patient has just experienced his “growth spurt” and is having some troubles with acne. He denies having any pain but admits that other children make fun of him at school. On examination, you note multiple large, skin-colored, pedunculated tumors. He also has evidence of acne, and there is a tan, oval macule measuring 3 cm on his chest. You believe he may have neurofibromatosis-1 (NF-1, also known as von Recklinghausen disease).

Which of the following statements about neurofibromatosis is false?

A. There are two major forms, NF-1 and neurofibromatosis-2 (NF-2)
B. Both NF-1 and NF-2 are inherited in an autosomal recessive pattern
C. NF-1 is characterized by neurofibromas, café au lait spots, iris hamartomas (Lisch nodules), neurologic impairment, and bone abnormalities
D. NF-2 is less common and is characterized by bilateral acoustic neuromas; skin findings are less common than in NF-1

Key Concept/Objective: To understand the key features of the two major forms of neurofibromatosis and their pattern of inheritance

It is most likely that this patient has NF-1 (von Recklinghausen disease) and that he inherited it from his father. Neurofibromas typically appear at puberty and are progressive, as are the other manifestations of neurofibromatosis. Such manifestations include café au lait spots; Lisch nodules; involvement of the spine and peripheral nerves with tumors; neurologic impairment; bony abnormalities; and a predisposition to malignancy. NF-2 is characterized by bilateral acoustic neuromas; NF-2 usually presents as hearing loss in the second or third decade of life. This form of neurofibromatosis is less common than NF-1 and is less likely to present with skin findings. Other features of NF-2 include meningiomas, gliomas, and cataracts. Both of these forms of neurofibromatosis are inherited in an autosomal dominant fashion, with near complete penetrance. (Answer: B—Both NF-1 and NF-2 are inherited in an autosomal recessive pattern)

50. A 17-year-old African-American adolescent presents with swelling of her earlobes; she had them pierced a few months ago. She can no longer put her earrings on and is distraught about her appearance. She notes that the involved areas itch and burn. Examination is remarkable for hyperpigmented, shiny, smooth tumors measuring 1 to 2 cm that are located around the areas of her ear piercing. There are small, crablike extensions from the lesions. There is no evidence of erythema or purulence. You believe the patient has developed keloids at the sites of her ear piercing.

Which of the following statements about keloids is false?

A. They are more common in African Americans, Hispanics, and those with a family history of keloids
B. Cryotherapy is ineffective
C. Intralesional steroids can flatten the keloids
D. Risk factors for the development of keloids include wound tension, ear piercing, healing by second intention, young age, and deep laceration

Key Concept/Objective: To understand the presentation of keloids, to know those who are at greatest risk for developing them, and to be aware of some of the common methods of treatment

Keloids represent an abnormal response to tissue injury, manifested as delayed, excessive proliferation of scar tissue. They do not regress and often cause pain, burning, and pruritus. They are more common in African Americans, Hispanics, and those with a family history of keloids. Risk factors for their development include wound tension, ear piercing, healing by second intention, young age, and deep laceration. Intralesional steroids administered at doses of 10 to 40 mg/ml every month for up to 6 months have been shown to effectively flatten keloids, although several side effects may occur. Cryotherapy given as a 30-second application once a month for 3 months has been found to be a safe and effective treatment. (Answer: B—Cryotherapy is ineffective)

For more information, see Abel EA: 2 Dermatology: XI Benign Cutaneous Tumors. ACP Medicine Online (www.acpmedicine.com). Dale DC, Federman DD, Eds. WebMD Inc., New York, July 2002

Acne Vulgaris and Related Disorders

51. A 16-year-old female patient comes to your office complaining of acne, which she has had for 3 years. The lesions have been small in size, not painful, and not swollen, and they have not progressed over this period. She says the acne is bothering her, and she would like to be treated. On physical examination, the patient is found to have multiple comedones measuring 0.5 to 1 mm that are open and closed on the face. Her arms, chest, and shoulders are not involved. There are no inflammatory lesions and no cysts. She is not sexually active.

Which of the following is the most appropriate treatment for this patient?

A. Educate the patient about diet and about trying to avoid chocolate and fatty meals
B. Start oral doxycycline
C. Start topical retinoids
D. Start oral isotretinoin
E. Start oral contraceptives

Key Concept/Objective: To know the appropriate treatment of comedonal acne

Comedones consist of keratinized cells and sebum. Comedonal acne consists of a predominance of open and closed comedones, without inflammatory findings such as erythematous papules, pustules, nodules, or cysts. The treatment for this form of acne should be directed toward improving the abnormal follicular keratinization process. The best option is topical retinoids, such as tretinoin or adapalene. Also, comedolytic agents such as salicylic acid may be used. Oral agents are not indicated in this mild and noninflammatory form of the disease. There is no role for dietary change in the management of acne. Oral contraceptives can be beneficial for patients with acne; these agents are ideal in women who are seeking birth control methods and in women who are not candidates for or have not responded to other treatments. (Answer: C—Start topical retinoids)

52. A 23-year-old man has a 5-year history of severe acne with scarring. His acne involves the face, shoulders, and chest. He has been treated in the past with multiple courses of topical agents, including retinoids, benzoyl, topical antibiotics, and oral antibiotics for 1 year. His lesions have not improved significantly through these agents. On physical examination, the patient has multiple large cysts and abscesses that are confluent and form sinus tracts.

Which of the following options are indicated in the management of this patient?
A. Change the oral antibiotic being used, because the presence of a resistant organism is very likely
B. Refer to a physician who is authorized to administer oral isotretinoin to consider starting this therapy
C. Start antiandrogenic therapy (e.g., spironolactone)
D. Perform a fungal culture of the lesions to exclude Malassezia folliculitis
E. Reassure the patient that acne is a disease of adolescents and that his symptoms should improve in the next few months

Key Concept/Objective: To know the indications for oral isotretinoin

Acne conglobata is a severe, scarring form of acne, with cysts, abscesses, and sinus tracts. This form of acne responds poorly to topical agents or oral antibiotics alone. Intralesional injections of corticosteroids and drainage of the abscesses are temporarily helpful, but these patients usually need oral isotretinoin for lasting improvement. Oral isotretinoin is the most effective agent available for the treatment of acne. It results in long-lasting remissions or cures in the majority of patients treated; however, it can produce severe side effects, so it should be used for severe forms of the disease only. To be able to prescribe and administer isotretinoin, physicians and pharmacists must be authorized and registered by the manufacturers of isotretinoin. Malassezia folliculitis does not respond to typical acne therapies with erythematous acneiform papules; however, it usually affects the extremities and is not characterized by the presence of abscess or sinus tracts. The resistance of Propionibacterium acnes to antibiotics has been well documented; however, antibiotic resistance is uncommon when benzoyl peroxide is used concomitantly with antibiotics. Furthermore, the severe form of acne seen in this patient usually does not respond to topical or antibiotic therapy. (Answer: B—Refer to a physician who is authorized to administer oral isotretinoin to consider starting this therapy)

53. A 21-year-old woman presents to the emergency department complaining of severe abdominal pain, which she has been experiencing for 2 days. The pain is epigastric and is accompanied by nausea and vomiting. Her medications include isotretinoin and oral contraceptives. She is sexually active and says she is using condoms in addition to oral contraceptives. On physical examination, the patient has moderately severe acne on her face and chest, and she has cheilitis. Her abdominal examination is remarkable for tenderness in the epigastric area and decreased bowel sounds.

Of the following, which one is the most likely diagnosis?
A. Ectopic pregnancy
B. Peptic ulcer disease
C. Ovarian cyst torsion
D. Acute pancreatitis
E. Appendicitis

Key Concept/Objective: To know the side effects of isotretinoin

Isotretinoin use is associated with important side effects. Some of the reported side effects are cheilitis, dryness of mucous membranes and skin, myalgias, pseudotumor cerebri, and hypercholesterolemia. Triglyceride levels can rise significantly: enough to cause acute pancreatitis, which is the most likely diagnosis in this case. Because teratogenicity occurs with even a single dose of isotretinoin, patients should undergo pregnancy testing repeatedly and should use two different methods of contraception. There have been reports of depression and suicide in isotretinoin-treated patients. This association remains controversial, but this risk must be discussed with the patient before starting therapy. (Answer: D—Acute pancreatitis)

54. A 16-year-old female patient comes to your office complaining of pimples. She states that the pimples appeared on her face 2 to 3 months ago. She has also noticed some deepening of her voice and the appearance of an increasing amount of hair on her chin and breasts. Her menses were regular until 4 or 5 months ago, when she started noticing irregular and short-lasting periods. On physical examination, the patient has papules and pustules on her face. Hirsutism is noted on her face, arms, breasts, and infraumbilical area, and she has an enlarged clitoris.

Which of the following is the most appropriate step to take next in the treatment of this patient?

A. Evaluate the patient for the possibility of ovarian or adrenal tumors
B. Start oral contraceptives for presumed polycystic ovarian syndrome (PCOS) and provide reassurance
C. Start low-dose hydrocortisone for presumed late-onset congenital adrenal hyperplasia and follow up in 2 months
D. Start benzoyl peroxide with topical metronidazole in the morning and topical retinoids at night
E. Administer a pregnancy test and consider isotretinoin therapy

Key Concept/Objective: To know the secondary causes of acne

Acne is very common in the adolescent population; however, it is important to recognize acne as a secondary manifestation of a primary process. Acne can be caused by medications such as isoniazid, corticosteroids, phenytoin, lithium, and progestins. Acne can also be a manifestation of androgen excess; this patient had acne of rapid onset, and there were other manifestations of androgen excess, such as changes in her voice, oligomenorrhea, clitoromegaly, and hirsutism. This rapid onset is characteristic of tumors producing androgens, which are frequently located in the ovarian or adrenal glands. Levels of testosterone, free testosterone, and dehydroepiandrosterone sulfate should be measured; if the results are high, imaging studies are indicated. PCOS can cause oligomenorrhea, hirsutism, and acne; however, it should not cause clitoromegaly or voice changes. Late-onset congenital adrenal hyperplasia (CAH)—21-hydroxylase deficiency in particular—is in the differential diagnosis. However, it would be atypical for it to present in such a rapid manner. If the workup for the presence of a tumor is negative, further workup for CAH would be indicated. (Answer: A—Evaluate the patient for the possibility of ovarian or adrenal tumors)

For more information, see Lebwohl M: 2 Dermatology: XII Acne Vulgaris and Related Disorders. ACP Medicine Online (www.acpmedicine.com). Dale DC, Federman DD, Eds. WebMD Inc., New York, November 2002

Disorders of Hair

55. A 38-year-old man presents to your clinic complaining of hair loss. He reports that the hair loss began several years ago but is more noticeable now. His hair has been thinning out on the sides as well as on the top of his head. He reports that several family members have the same problem.

Which of the following is the most appropriate therapy for this patient’s condition?

A. Topical 2% minoxidil applied to the scalp twice daily
B. Mediumto high-potency topical steroids applied daily
C. Topical immunotherapy with the sensitizing chemical diphencyprone
D. Short-contact topical therapy with 0.25% anthralin cream applied daily

Key Concept/Objective: To be able to recognize and appropriately treat androgenetic alopecia

Androgenetic alopecia is the most common type of nonscarring hair loss affecting the crown. It results from a genetically determined end-organ sensitivity to androgens. It is often referred to as common baldness, male-pattern alopecia, and female-pattern alopecia. Androgenetic alopecia affects at least 50% of men by 50 years of age and 50% of women by 60 years of age. Men have more androgen than women and therefore are usually affected earlier and more severely. Male-pattern alopecia often starts in persons between 15 and 25 years of age. Male-pattern alopecia has two characteristic components, bitemporal recession and vertex balding, which in pronounced cases can progress to complete balding of the crown. Depending on the severity of the condition, management of androgenetic alopecia ranges from watchful inactivity to medical and surgical treatment; a hairpiece or wig may be used in the most refractory cases. Minoxidil is applied twice daily with a dropper, spread over the top of the scalp, and gently rubbed in. The drug should be tried for at least a year. Minoxidil acts by initiating and prolonging anagen. It produces visible hair growth in approximately one third of male and female patients, fine-hair growth in approximately one third, and no growth in approximately one third. It is more effective as a preventive agent, retarding hair loss in approximately 80% of patients. Topical steroid therapy, topical immunotherapy, and use of anthralin cream are therapeutic choices for the treatment of alopecia areata. (Answer: A—Topical 2% minoxidil applied to the scalp twice daily)

56. A 42-year-old woman presents to the walk-in clinic complaining of hair loss. She notes the hair loss mostly on the top of her head but also on the sides. She reports a history of hypertension and diabetes but is currently on no medications. On review of systems, she reports some menstrual irregularities but is otherwise without complaint. She has tried some over-the-counter products for facial hair growth, but they have not helped the hair loss on her scalp. She would like to try something “prescription strength.”

Of the following, which is the most appropriate step to take next in the treatment of this patient?
A. Begin therapy with topical 2% minoxidil applied twice daily and see her again in 3 to 6 months
B. Begin daily therapy with ethinyl estradiol-ethynodiol diacetate and see her again in 6 weeks
C. Explain the risks and benefits of hormone replacement therapy for perimenopausal symptom relief and see her again in 1 to 2 months
D. Screen the patient for hyperandrogenism and see her again in 1 to 2 weeks

Key Concept/Objective: To be able to differentiate simple androgenetic alopecia from hyperandrogenism

The constellation of symptoms seen in this patient should raise concern about the possibility of excess circulating androgen. The diagnosis of androgenetic alopecia is usually obvious from the clinical pattern of hair loss from the top of the head. In some men, a female pattern of alopecia causes diagnostic confusion but has no other significance. In women, a male pattern of alopecia (i.e., bitemporal recession and vertex balding) in association with menstrual irregularities, acne, hirsutism, and a deep voice is significant. The virilism indicates significant hyperandrogenism, the cause of which must be identified and treated. Local therapy with either topical minoxidil or ethinyl estradiol-ethynodiol diacetate is reasonable for women with a diagnosis of androgenetic alopecia; however, in this case, the cause of virilism should be thoroughly investigated before initiating any therapy. (Answer: D—Screen the patient for hyperandrogenism and see her again in 1 to 2 weeks)

57. A 32-year-old woman presents to your office complaining of hair loss. The hair loss is occurring all over her head and seems to spare no area. She notes that it worsens when she showers, and she has begun showering every other day in an attempt to decrease the hair loss. She has been healthy all her life, and other than an uneventful pregnancy and vaginal delivery 3 months ago, she has no medical history.

This patient’s clinical presentation and history are most consistent with which of the following hair disorders

A. Androgenetic alopecia
B. Telogen effluvium
C. Alopecia areata
D. Cicatricial alopecia

Key Concept/Objective: To know the clinical presentation of telogen effluvium

Telogen effluvium is the most common form of diffuse alopecia. It presents as a generalized shedding of telogen hairs from normal resting follicles. The basic cause of telogen effluvium is a premature interruption of anagen, which leads to an increase in the number of hairs phased into telogen. When the 3-month telogen period ends, new anagen hairs grow in and numerous telogen hairs fall out. Patients may need reassurance that this apparent loss of hair is actually a sign of regrowth. Acute telogen effluvium can be caused by childbirth, febrile illnesses, surgery, chronic systemic diseases, crash diets, traction, severe emotional stress, and drug reactions. It can also be a physiologic reaction in neonates. During acute telogen effluvium, pull tests are positive all over the scalp, yielding two to 10 club hairs. Telogen effluvium is often accompanied by bitemporal recession; this is a useful diagnostic sign in women. The acute form usually ends within 3 to 6 months. The diagnosis is usually made on the basis of the history of an initiating event 3 months before the onset of shedding. No treatment is needed for acute telogen effluvium, because the hair invariably regrows within a short time. Alopecia areata is characterized by patchy areas of hair loss, not the diffuse hair loss seen in this patient. The pattern of hair loss in this female patient is not consistent with androgenetic alopecia (thinning of hair in the crown). The lack of skin changes makes the possibility of cicatricial alopecia remote. (Answer: B—Telogen effluvium)

For more information, see Whiting DA: 2 Dermatology: XIII Disorders of Hair. ACP Medicine Online (www.acpmedicine.com). Dale DC, Federman DD, Eds. WebMD Inc., New York, January 2003

Diseases of the Nail

58. A 36-year-old man comes to your clinic for the first time for a check-up. He has no active complaints. Review of systems is negative. His medical history is significant only for hypertension. He neither smokes nor drinks. His only medication is hydrochlorothiazide. Physical examination is unremarkable except for the presence of digital clubbing bilaterally. You asked the patient about this finding, and he says his nails have always looked like this and that his father had similar nails.

What would be an appropriate workup for this patient?
A. Obtain a chest x-ray to rule out intrathoracic pathology
B. Obtain an echocardiogram to rule out congenital heart disease
C. Order hand x-rays to further document the presence of clubbing
D. No further workup is necessary

Key Concept/Objective: To understand the differential diagnosis of clubbing

When the normal angle between the proximal nail fold and the nail plate exceeds 180°, digital clubbing is present. The morphologic changes of clubbing typically include hypertrophy of the surrounding soft tissue of the nail folds as a result of hyperplasia of dermal fibrovasculature and edematous infiltration of the pulp tip. Clubbing may be hereditary, or it may be seen in association with several underlying disease states, such as hypertrophic pulmonary osteoarthropathy, chronic congestive heart failure, congenital heart disease associated with cyanosis, polycythemias associated with hypoxia, Graves disease, chronic hepatic cirrhosis, lung cancer, and Crohn disease. This patient has no signs of diseases associated with clubbing, he has had clubbing for many years, and he has a family history of clubbing; therefore, further workup is not indicated. When clubbing is unilateral, consideration should be given to underlying causes of impaired circulation. (Answer: D—No further workup is necessary)

59. A 22-year-old woman presents to a walk-in clinic complaining of pain and swelling on one of her fingers. The swelling and pain are located on the second finger of her right hand, just proximal to her nail. She has also noticed some pus coming from this area. The symptoms started 3 days ago. She reports that she has been cutting her cuticle constantly for cosmetic reasons. Examination reveals erythema, swelling, and purulence of the nail fold of her second finger, and the area is very tender to palpation.

What is the appropriate treatment for this patient’s condition?
A. Drainage of the focal abscess and administration of oral antibiotics active against Staphylococcus aureus
B. Oral fluconazole therapy for presumed Candida paronychia
C. Administration of steroid cream for presumed contact dermatitis affecting the nail fold
D. Hand x-ray to rule out osteomyelitis

Key Concept/Objective: To understand the differential diagnosis and management of paronychia

The nail folds are the cutaneous soft tissue that houses the nail unit, invaginating proximally and laterally to encompass the emerging nail plate. The term paronychia denotes inflammation of the nail folds. Paronychia may be acute or chronic and may occur secondary to a variety of conditions, including contact dermatitis, psoriasis, bacterial infections, and fungal infections. The cuticle is a thin, keratinized membrane that serves as a seal to protect the nail fold from exposure to external irritants, allergens, and pathogens. Bacterial paronychia is usually acute in nature. It is characterized by swelling, erythema, discomfort, and sometimes purulence. The most common etiologic pathogen is Staphylococcus aureus. Treatment requires drainage of a focal abscess, if present, and oral antibiotic therapy. Chronic paronychia results from chronic irritant dermatitis and loss of cuticle from trauma or nail-care practices; it also occurs secondary to candidal infection. (Answer: A—Drainage of the focal abscess and administration of oral antibiotics active against Staphylococcus aureus)

60. A 33-year-old man comes to your clinic complaining of weight loss. He has also been experiencing occasional diarrhea. He started to have these symptoms 4 months ago. He says he has been trying to eat more, but he is still losing weight. Physical examination shows bitemporal wasting, diffuse cervical lymphadenopathy, and proximal white subungual lesions. These lesions show dermatophytes on potassium hydroxide (KOH) staining.

What is the most likely diagnosis for this patient?
A. Graves disease
B. HIV infection
C. Lymphoma
D. Inflammatory bowel disease

Key Concept/Objective: To know the clinical presentation of white proximal onychomycosis

Onychomycosis, the most common infection of the nail, is a fungal infection characterized by nail-bed and nail-plate involvement. Dermatophyte onychomycosis is the most common type of fungal nail infection. The most characteristic clinical features of dermatophyte onychomycosis are distal onycholysis, subungual hyperkeratosis, and a dystrophic, discolored nail plate. Because this combination of features is also seen in persons with nail psoriasis, accurate diagnosis may require KOH preparation and fungal culture. The clinical presentation of proximal white subungual onychomycosis has been reported in association with systemic immunosuppression, including immunosuppression associated with HIV disease. The presence of this pattern of onychomycosis should prompt an evaluation for HIV disease. This patient has a very high probability of having HIV infection, given the clinical presentation. Candida onychomycosis is far less common than dermatophyte onychomycosis and is also often associated with immunosuppression. (Answer: B—HIV infection)

For more information, see Del Rosso JQ, Daniel CR III: 2 Dermatology: XIV Diseases of the Nails. ACP Medicine Online (www.acpmedicine.com). Dale DC, Federman DD, Eds. WebMD Inc., New York, February 2003

Disorders of Pigmentation

61. A 34-year-old African-American woman comes to your clinic for evaluation of dark spots on her face.
These dark patches have been appearing over the past 2 or 3 months. She denies having any itching or other symptoms in this area. She has no history of similar skin lesions in the past. The patient has just delivered a healthy baby girl by cesarean section. She has no significant medical history; her only medication is a multivitamin. Physical examination shows hyperpigmented patches in the malar region bilaterally. The rest of her skin examination is normal.

What is the most likely diagnosis, and how would you treat this patient?
A. Lentigines; start hydroquinone
B. Melasma; start a sunscreen and hydroquinone
C. Postinflammatory hyperpigmentation; start azelaic acid
D. Vitiligo; start topical corticosteroids

Key Concept/Objective: To know the clinical picture and treatment of melasma

Melasma is a common acquired symmetrical hypermelanosis characterized by irregular light-brown to gray-brown macules involving the face. Melasma is commonly observed in females; men constitute only 10% of the cases. It occurs more commonly in geographic regions that receive intense ultraviolet radiation, such as tropical or subtropical regions. Clinically, the light-brown patches are commonly evident on the malar prominences, forehead, chin, nose, and upper lip. The patches may have a centrofacial or mandibular distribution. Current treatments for melasma include broad-spectrum sunscreens, hydroquinone formulations, azelaic acid, kojic acid, a-hydroxy acid products, retinoic acid, retinol, superficial chemical peels, and microdermabrasion. Although all these therapies improve melasma, none are curative. It is essential for patients to adhere to a regimen of daily sun protection. A lentigo is a well-circumscribed, brown-black macule that appears at birth or early childhood. Postinflammatory hyperpigmentation is characterized by an acquired increase in cutaneous pigmentation secondary to an inflammatory process; there is no such history in this patient. Vitiligo is a common skin disorder characterized by one or more patches of depigmented skin. (Answer: B—Melasma; start a sunscreen and hydroquinone)

62. A 16-year-old white male comes to your clinic for evaluation of skin pigmentation. For the past 6 weeks, he has been experiencing progressive dark pigmentation in both arms. He denies having had such an illness in the past. The patient has acne, which was first diagnosed 2 years ago. His acne affects his face and upper back. He says he is not taking any medications except for an “acne pill.” Physical examination is remarkable for the presence of comedones and pustules on his face and upper back. The patient also has hyperpigmented skin in both arms. No other skin lesions are present; the rest of the examination is normal.

Which of the following is the most likely diagnosis for this patient?
A. Postinflammatory hyperpigmentation
B. Addison disease
C. Erythema dyschromicum perstans
D. Drug-induced hyperpigmentation

Key Concept/Objective: To recognize hyperpigmentation as a possible side effect of different medications

Medications are a common cause of cutaneous hyperpigmentation. Lesions may be localized or generalized. Medications can also cause hyperpigmentation of the oral mucosa and nails. Medications causing drug-induced hyperpigmentation include oral contraceptives, hormone replacement therapies, antibiotics, antidepressants, antiviral agents, antimalarials, antihypertensives, and chemotherapeutic agents. This patient has acne, for which he is taking medication. This medication could be minocycline or tetracycline—antibiotics that are commonly used for treatment of acne and that can cause hyperpigmentation. This makes drug-induced hyperpigmentation the most likely etiology. Postinflammatory hyperpigmentation can be a sequela of several dermatologic conditions, including acne; however, the hyperpigmentation in this patient is not located where the acne lesions are. Addison disease can cause hyperpigmentation on the skin and mucosal surfaces; however, this patient has no other signs and symptoms of this disorder. Erythema dyschromicum perstans is an acquired benign condition characterized by the presence of slate-gray to violaceous macules. The lesions are usually symmetrically distributed and vary in size from small macules to very large patches. Common sites of involvement include the face, neck, trunk, and upper extremities. This patient’s clinical picture is not consistent with this disorder. (Answer: D—Drug-induced hyperpigmentation)

63. A 40-year-old African-American woman with a history of hypertension comes to your clinic for a followup visit. She is in her usual state of health. Her only medication is hydrochlorothiazide. Physical examination reveals an area of hypopigmented skin measuring 5 × 7 cm on her right foot.

Which of the following is the most likely diagnosis for this patient, and what further workup is indicated?
A. Albinism; skin biopsy
B. Vitiligo; no further workup is indicated
C. Vitiligo; complete blood count, sedimentation rate, comprehensive metabolic panel, and autoantibody tests
D. Idiopathic guttate hypomelanosis; skin biopsy

Key Concept/Objective: To know the clinical picture and appropriate work-up of vitiligo

Vitiligo is a common acquired, idiopathic skin disorder characterized by one or more patches of depigmented skin caused by loss of cutaneous melanocytes. Onset may begin at any age, but peak incidences occur in the second or third decade of life. There is no racial predilection; females are affected more often than males. Vitiliginous lesions are typically asymptomatic depigmented macules without signs of inflammation. Occasionally, they may show signs of inflammation or pruritus. Areas of depigmentation vary in size from a few millimeters to many centimeters. In view of the association of vitiligo with myriad other autoimmune diseases, the routine evaluation of a patient should include a thorough history and physical examination. Recommended laboratory tests include a complete blood count, sedimentation rate, comprehensive metabolic panel, and autoantibody tests (antinuclear antibody, thyroid peroxidase, and parietal cell antibodies). Albinism is an uncommon congenital disorder characterized by hypopigmentation of the hair, eyes, and skin. Idiopathic guttate hypomelanosis is a common asymptomatic disorder characterized by hypopigmentation and depigmented polygonal macules ranging from approximately
2 to 8 mm in diameter. (Answer: C—Vitiligo; complete blood count, sedimentation rate, comprehensive metabolic panel, and autoantibody tests)

64. A 1-year-old boy is being evaluated for recurrent pneumonia. His history includes three episodes of pneumonia and one episode of skin infection. His family history is unremarkable. Physical examination shows ocular hypopigmentation; there are also two areas of skin hypopigmentation, one on his face and one on his left arm. His peripheral blood smear is remarkable for the presence of giant cytoplasmic granules in the neutrophils.

Which of the following is the most likely diagnosis for this patient?
A. Chédiak-Higashi syndrome
B. Prader-Willi syndrome
C. Hermansky-Pudlak syndrome
D. Cross-McKusick-Breen syndrome

Key Concept/Objective: To know the classic features of Chédiak-Higashi syndrome

Chédiak-Higashi syndrome is characterized by recurrent infections, peripheral neuropathy, and oculocutaneous hypopigmentation. This disorder leads to death at an early age as a result of lymphoreticular malignancies or infections. The presence of giant lysomal granules in the neutrophils is characteristic of Chédiak-Higashi syndrome. Prader-Willi syndrome is a developmental syndrome characterized initially by mental retardation, neonatal hypotonia, and poor feeding, followed by hyperphagia and obesity. Patients have ocular abnormalities and skin and hair hypopigmentation consistent with oculocutaneous albinism. Hermansky-Pudlak syndrome presents as a hemorrhagic diathesis. Skin and hair color varies from white to light brown. Freckles and lentigines develop with age. CrossMcKusick-Breen syndrome is characterized by hypopigmentation, microphthalmia, nystagmus, and severe mental and physical retardation. (Answer: A—Chédiak-Higashi syndrome)

65. A 22-year-old woman requests birth control pills. She has just moved to the United States from Poland with her new husband. She has no history of illness or current illness and has not seen a doctor in a long time. On examination, multiple flat, brown, uniformly pigmented 1 to 3 cm macules, as well as several fleshy, almost pedunculated, nodules are seen on her left leg, hip, and buttock. These lesions stop abruptly at midline on her back. None of these lesions are seen elsewhere on her body. She recalls having these all her life.

Which of the following steps will be useful in the management of this patient’s skin lesions?
A. No further interventions are required
B. Referral for genetic counseling
C. Ophthalmologic screening
D. Diagnostic skin biopsy
E. Topical steroid therapy

Key Concept/Objective: To recognize segmental neurofibromas and café au lait spots as the result of a focal mutation (type 5 neurofibromatosis) that is not associated with systemic or heritable disease

Type 5 neurofibromatosis manifests as skin lesions in a focal dermatomal segment caused by a localized mutation occurring during embryonic development. It does not carry the risk of the CNS tumors, cortical cataracts, or multiorgan system involvement seen with other types of neurofibromatosis. Additionally, focal dermatomal mutations do not affect the germline cells and are therefore not heritable, in contrast to the autosomal dominant pattern of inheritance of the other neurofibromatosis disorders. Therefore, there is no need for genetic counseling. Although ophthalmologic screening can confirm neurofibromatosis type 1 by finding Lisch nodules (iris hamartomas), this screening is not necessary in patients with type 5 neurofibromatosis because there is no associated eye disorder. Diagnosis of type 5 neurofibromatosis is made by inspection of the skin lesions, and no biopsy is required. Topical steroids have no effect on café au lait spots or neurofibromas. (Answer: A—No further interventions are required)

66. A 5-year-old girl is brought to the office by her mother after a fall that caused a forehead laceration. The fall was unwitnessed and occurred during the care of a babysitter. The child has been in good health, though she has been slower than other children her age in reaching the usual milestones in cognitive development. On examination, the patient is a fair-skinned, dark-haired child with a streak of gray hair on the left scalp. She has a 2 cm laceration on her forehead, with considerable associated swelling. Several papules are apparent on her face. A Wood lamp examination reveals four pale, elongated macules on her trunk.

Which disorder best accounts for the findings in this case?
A. Neurofibromatosis
B. Pityriasis alba
C. Vitiligo
D. Piebaldism
E. Tuberous sclerosis

Key Concept/Objective: To be able to recognize tuberous sclerosis by a finding of hypopigmented macules in the setting of likely mental retardation, seizures, and facial angiofibromas

Tuberous sclerosis presents in infants and young children with a classic triad of hypopigmented, ash-leaf-shaped macules (including those in the scalp, which cause gray streaks of hair), mental retardation, and seizures, though the last two can be absent. This child’s unwitnessed fall could have been a seizure, and her slowness to attain developmental milestones may be a sign of mental retardation. The finding of facial neurofibromas, skin-colored-to-red papules, is pathognomonic. In contrast, skin findings in neurofibromatosis include hyperpigmented, not hypopigmented, macules, as well as nodular neurofibromas in distributions other than the face. Pityriasis alba affects young children, usually those with dark skin. The lesions are usually on the face and may begin with subtle erythema and slightly raised borders. The lesions then become scaly, hypopigmented macules with indistinct borders and regress spontaneously after several months. Vitiligo causes depigmentation of skin and hair that develops in older children and young adults, usually those of darker skin color. Associated conditions include autoimmune endocrine disorders but not seizures or retardation. Piebaldism causes a white forelock in 90% of patients, as well as amelanotic macules on the trunk, extremities, and mucous membranes. These are present at birth and remain stable over time, unlike vitiligo, which develops later in life and is often progressive. (Answer: E—Tuberous sclerosis)

67. A 24-year-old Hispanic woman has been using oral contraceptives and was treated with ciprofloxacin for a bladder infection several months ago. She presents with concerns about some spots on her face that she would like to have removed. The spots appeared suddenly over the past few weeks. They are not itchy or painful. On examination, the patient has blotchy, hyperpigmented, brown macules over the central face, without scaling or induration, involving the nose, nasolabial folds, upper lip, cheeks, and forehead. There are no lesions on the oral mucosa and no rashes elsewhere on her body.

What is the most likely cause of this patient’s hyperpigmented lesions?
A. Melasma
B. Tinea faciei
C. Lichen planus
D. Lupus erythematosus
E. Drug-induced sun sensitivity

Key Concept/Objective: To recognize melasma in patients with risk factors and characteristic skin findings

Melasma causes hyperpigmented macules in the central areas of the face. Risk factors for melasma include dark skin, female gender, oral contraceptive use, pregnancy, and sun exposure. Tinea faciei can cause hyperpigmented lesions on the face, but these are usually scaly, with annular accentuation of hyperpigmentation and central clearing. Lichen planus is usually quite itchy and usually occurs in locations such as the wrists, back, shins, and buccal mucosa, but it can also involve the eyelids, tongue, lips, or scalp. Lichen planus is usually more violaceous in color and contains fine, parallel, lacy white lines called Wickham striae. Lupus can cause a malar rash, which is usually erythematous rather than brown and usually confluent rather than blotchy, with some associated fine scaling. Like melasma, this classic malar rash can worsen with sun exposure, but it spares the nasolabial folds. Drug-induced sun sensitivity should also spare the nasolabial folds and upper lip because these areas receive less sun exposure than do other areas of the face. (Answer: A— Melasma)

For more information, see Grimes PE: 2 Dermatology: XV Disorders of Pigmentation. ACP Medicine Online (www.acpmedicine.com). Dale DC, Federman DD, Eds. WebMD Inc., New York, February 2003